Download Drug Therapy

Document related concepts

Adherence (medicine) wikipedia , lookup

Psychedelic therapy wikipedia , lookup

Orphan drug wikipedia , lookup

Stimulant wikipedia , lookup

Compounding wikipedia , lookup

Discovery and development of proton pump inhibitors wikipedia , lookup

Polysubstance dependence wikipedia , lookup

Discovery and development of beta-blockers wikipedia , lookup

Bad Pharma wikipedia , lookup

Drug design wikipedia , lookup

Pharmacognosy wikipedia , lookup

Drug discovery wikipedia , lookup

Psychopharmacology wikipedia , lookup

Medication wikipedia , lookup

Pharmaceutical industry wikipedia , lookup

Prescription drug prices in the United States wikipedia , lookup

Pharmacokinetics wikipedia , lookup

Neuropsychopharmacology wikipedia , lookup

Prescription costs wikipedia , lookup

Pharmacogenomics wikipedia , lookup

Drug interaction wikipedia , lookup

Neuropharmacology wikipedia , lookup

Transcript
Module. Drug Therapy.
Text test questions:
1 The drug applied at the case of hypersusceptibility to nitroglycerine for removing attack of angina
pectoris
A. Aspirin
B. Atropine
C. Piracetame
D. Papaverine
E. * Molsidomine
2 All of the following statements concerning nitroglycerin are correct EXCEPT:
A. It causes an elevation of intracellular cGMP.
B. It undergoes significant first-pass metabolism in the liver.
C. It may cause significant reflex tachycardia.
D. * It significantly decrease AV conduction.
E. It can cause postural hypotension.
3 The clot aggregation blocker oppressing the adenosine absorption and reducing its enzyme destruction
A. Papaverine
B. Propranolol
C. Reserpine
D. * Dipiridamole
E. Trental
4 What group does Trimetazidine belong to?
A. Adrenomimetics
B. Cholinoblockers
C. * Cardioprotectors
D. Sympatholitics
E. Diuretics
5 The cardioselective beta-adrenoblocker
A. * Bisoprolol
B. Adrenalin
C. Riboxinum
D. Enalapril
E. Nifedipine
6 The drug applied for the medical treatment of ischemic heart disease
A. Sympatholitics
B. * beta-adrenoblockers
C. alpha-adrenoblockers
D. Myorelaxants
E. M-cholinomimetics
7 Which of the following adverse effects is associated with nitroglycerin?
A. Hypertension
B. * Throbbing headache
C. Bradycardia
D. Sexual dysfunction.
E. Anemia.
8 Which drug belongs to the beta-adrenoblockers’ group?
A. Nitroglycerine
B. * Metoprolol
C. Captopril
D. Nifedipine
E. Furosemidum
9 All of the following mechanisms of action correctly match a drug EXCEPT:
10
11
12
13
14
15
16
17
A. Quinidine: Blocks Na+ channels
B. Brethylium: Blocks K+ channels
C. Verapamil: Blocks Ca++ channels
D. Propranolol: Blocks beta-adrenoreceptors
E. * Procainamide: Blocks K+ channels.
The action of nitroglycerine and others organic nitrates is caused by
A. The blockade of potassium channels
B. The blockade of sodium channels
C. The blockade of calcium channels
D. * The release of nitrogen oxyde in the vessel walls
E. The excitation of beta2 -adrenoreceptors vessels
The efficiency of Nifedipine at arterial hypertension is caused by
A. * The dilation of vessels
B. The rise of myocardium contractility
C. The decline of renine production
D. The decline of diuresis
E. The rise of diuresis
Which drug belongs to the Calcium-channel blocker group?
A. Papaverinum
B. Atenolol
C. * Verapamil
D. Furosemidum
E. Anaprilinum
The drug applied for the medical treatment of ischemic heart disease
A. alpha-adrenoblockers
B. Sympatholitics
C. Diuretics
D. M-cholinomimetics
E. * Nitrates
Which one of the following statements is INCORRECT?
A. Lidocaine must be given parenterally.
B. * Lidocaine is used mainly for atrial arrhythmias.
C. Procainamide is associated with a reversible lupus phenomenon.
D. Quinidine is active orally.
E. All antiarrhythmic drugs can suppress cardiac contractions.
The basic principle of Nifedipine action
A. It rises the activity of potassium channels
B. It blockades beta2-adrenoreceptors
C. The release of nitrogen oxide
D. * It blockades the calcium channels L-type
E. It blockades beta1 -adrenoreceptors
The basic effect of nitroglycerine
A. Strengthening diuresis
B. * Dilation of arterioles and venules
C. Oppression of heart contraction
D. Anti-inflammatory action
E. Rise of myocardium contractility
The drug related to the group of calcium antagonists
A. Reserpine
B. Apressinum
C. * Nifedipine
D. Prazosinum
E. Papaverini hydrochloridum
18 Which drug decreases the demand in oxygen due to the blockade of the sympathetic innervation?
A. Molsidomin
B. * Atenolol
C. Pentoxiphylline
D. Papaverinum
E. Nitroglycerine
19 An efficiency of beta-adrenoblockers at angina pectoris is explained by
A. * The removal of the sympathetic influences on the heart because of the blockade of the betaadrenoreceptors
B. The expansion of the coronal vessels
C. The increase of sympathetic influences on the heart
D. The increase of cardiac abbreviations
E. The decline of thrombocyte aggregation
20 Which one of the following statements is INCORRECT?
A. Quinidine prolongs repolarization and the effective refractory period.
B. Mexiletine shortens repolarization and decreases the effective refractory period.
C. * Propranolol increases phase 4 depolarization.
D. Verapamil shortens the duration of the action potential.
E. Amiodarone prolongs repolarization.
21 The drug, applied sublingually at the attack of angina pectoris
A. Isosorbide dinitrate
B. Riboxinum
C. Pentoxiphylline
D. Atenolol
E. * Nitroglycerinum
22 The drug of the group of nitrates with the prolonged action at the ischemic heart disease
A. Validolum
B. Losartan
C. * Sustac
D. Magnesium sulfate
E. Captopril
23 Which drug belongs to the group of beta-adrenoblockers
A. Nitroglycerine, Molsidomine
B. * Nebivolol, Atenolol
C. Pentoxiphylline, Papaverinum
D. Nifedipine, Verapamil
E. Captopril, Enalapril
24 Which drug belongs to the group of calcium ions antagonists?
A. Molsidomine, Nitroglycerine
B. Anaprilinum, Atenolol
C. Pentoxiphylline, Papaverinum
D. * Amlodipine, Verapamil
E. Captopril, Enalapril
25 Which one of the following statements about antiarrhythmic drugs is CORRECT?
A. * They may act by converting unidirectional block to a bidirectional block.
B. They often cause an increase in cardiac output.
C. As a group they have mild side effects.
D. They all affect Na+ channels in the cell membrane.
E. They are equally useful in atrial and ventricular arrhythmias.
26 Which drug belongs to the inhibitors of ACE
A. Nitroglycerine, Molsidomine
B. Anaprilinum, Atenolol
C. Pentoxiphylline, Papaverinum
27
28
29
30
31
32
33
34
35
D. Nifedipine, Verapamil
E. * Lisinopril, Enalapril
Indicate the antiarrhythmic drug
A. Nitroglycerin
B. Isosorbiti dinitras
C. Sustac
D. Validolum
E. * Procainamide (Novocainamide)
Indicate the antiarrythmic drug which exerts membrane-stabilizing action.
A. * Quinidine sulfate
B. Isadrinum
C. Digoxin
D. Nitroglycerin
E. Atenolol
Indicate the antiarrytmic drug of adrenoblockers group
A. Amiodaron
B. * Metoprolol
C. Verapamil
D. Asparcam
E. Nitroglycerin.
Indicate the peculiarities of the action of quinidine upon heart.
A. * Increase of the effective refractory period.
B. Decrease of the effective refractory period.
C. Increase of the conduction of myocardium.
D. Inhibition of the conduction of myocardium.
E. Increase of the cardic automatism.
The major drawback to antianginal use of propranolol is:
A. * Exacerbation of congestive heart failure
B. Increased blood pressure
C. urine retention
D. blurred vision
E. diabetes-like hyperglycemia
Indicate the mechanism of verapamil action.
A. Blockade of beta-adrenoceptors of the heart.
B. * Blockade of calcium channels of the cardiomyocyte membranes.
C. Blockade of potasium channels of myocytes.
D. Inhibition of phosphodiesterase .
E. Inhibition cholinesterase.
Indicate the pharmacological effect of verapamil.
A. Cardiotonic effect.
B. Hypertensive effect.
C. * Antiarrhythmic effect.
D. Diuretic effect.
E. Antibacterial effect.
Which drug is used for ischemic heart disease, hypertension and cardiac arrhythmias?
A. * Bisoprolol
B. Sustac
C. Nitroglycerine
D. Novocainamidum
E. Validolum
To indicate the possible side effect of Propranolol?
A. * Bronchial spasm
B. Tachycardia
36
37
38
39
40
41
42
43
44
C. Rise of arterial pressure
D. Ortostatic collapse
E. Development of tolerance
Why Novocainamide can’t inject intravenously quickly?
A. Cause tachycardia
B. * Cause decreasing of arterial pressure
C. Cause hypertension
D. Depress respiration
E. Have short acting period
Ventricular premature depolarizations. What agent is indicated?
A. Isosorbide dinitrate
B. * Disopyramide
C. Propranolol
D. Nifedipine
E. Nitroglycerin
What antiarrhythmic drug is paraaminobenzioc acid derevate?
A. * Novocainamidum
B. Quinidinum
C. Verapamilum
D. Xycainum
E. Dipheninum
What antiarrhythmic drug has local anaesthetic activity?
A. Novocainamidum
B. Quinidinum
C. Verapamilum
D. * Xycainum
E. Dipheninum
For treatment of what arrythmias Lidocaine is the most effective?
A. * Ventrical tachyarrhythmia
B. Atrium tachyarrhythmia
C. Atrio-ventrical block
D. Sino-auricular block
E. Chronic paroxismal arrhythmia
For treatmant of what kind of arrythmia cardiac glycosides are contraindicated?
A. * Ventricula tachyarrhythmia
B. Atrium tachyarrhythmia
C. Extrasystolia
D. Flutering of atrium
E. Chronic paroxismal arrhythmia
Isosorbide dinitrate is used:
A. Antiarrhythmic action is via alpha-adrenergic blockade
B. Ventricular premature depolarizations
C. * Angina pectoris prophylaxis
D. Conversion of atrial fibrillation to sinus rhythm
E. Treatment of ventricular fibrillation
What medicine is a drug of choice for treatment of ventricular arrythmia?
A. Novocainamidum
B. Quinidinum
C. * Xycainum
D. Dipheninum
E. Verapamilum
When quinidine is administered to a patient with atrial fibrilation?
A. the ventricular rate may increase dangerously
45
46
47
48
49
50
51
52
53
B. arterial hypotension usually results if the drug is administered intravenously
C. thrombi attached to fibrillating atria may embolize
A. * all are correct
B. none are correct
To indicate medication of Nitroglycerine of prolong action
A. * Sustac
B. Amoidaronum
C. Molsidominum
D. Fenigidinum
E. Anaprilinum
Indicate the side effect of nitroglycerine
A. Itching of skin
B. Bradicardia
C. Rise of arterial pressure
D. * Headache
E. Intestine atony
Which antianginal drug provokes bronchial spasm?
A. Sustac
B. * Propranolol
C. Nitroglycerine
D. Fenigidinum
E. Molsidominum
Why during attack of angina pectoris Sustac is not prescribed orally?
A. Brief action
B. Causes tachycardia
C. * Long latent period
D. Causes hypotension
E. Causes tolerance
Indication to Sustac.
A. * Prophylaxis of attacks of angina pectoris
B. Removal of attacks of angina pectoris
C. Medical treatment of acute heart attack
D. Medical treatment of high blood pressure illness
E. Medical treatment of paroxismal tachycardia
Agents which may lower circulating plasma lipids include:
A. Clofibrate
B. Nicotinic acid
C. Cholestyramine
D. Dextrothyroxine
E. * All are correct
Which medication is drug of choice at attack of angina pectoris?
A. Sustac
B. Molsidominum
C. * Nitroglycerine
D. Anaprilinum
E. Fenigidinum
Which from the antianginal agents can be applying for the removal and for prophylaxis of attacks of
angina pectoris?
A. * Trinitrolong
B. Nitroglycerine
C. Sustac
D. Corvatonum
E. Validolum
54 Why is not it impossible to apply nitroglycerine in case of high intracranial pressure?
A. Increase arterial pressure
B. Causes bradycardia
C. * Increase intracranial pressure
D. Decrease intracranial pressure
E. Provoke angina pectoris
55 Increases transmembrane action potential duration:
A. Lidocaine
B. * Quinidine
C. Atenolol
D. Digoxin
E. Esmolol
56 Which from agents is used for ischemic heart disease and hypertension?
A. * Nifedipine
B. Sustac
C. Nitroglycerine
D. Novocainamaidum
E. Aspirine
57 Indicate possible side effect of propranolol?
A. Hypertension
B. Tachycardia
C. * Bradycardia
D. Оrtostatic collapse
E. Development of tolerance
58 The patient suffers on the ischemic heart disease. Which from the adopted antianginal drugs has coronary
dilation and antiplatelet action?
A. * Dipiridamolum
B. Acetylsalicylic acid
C. Validolum
D. Nitroglycerine
E. Molsidominum
59 To note the drug for “controlled hypotension”
A. Reserpinum
B. Anaprilinum
C. Methyldopa
D. * Hygronium
E. Sibasonum
60 Extensively metabolized by circulating enzymes:
A. Quinidine
B. Lidocaine
C. Nicotinic acid
D. Cholestyramine
E. * Neither
61 To indicate mechanism of Са2+ antagonists hypotensive action
A. Decrease the tonus of vasomotor centers
B. Block the postsynaptic beta-adrenoreceptors
C. Block the sympathetic ganglions
D. * Block the calcium channels
E. Stimulate the production of prostaglandins
62 Which one of the following is the most common side effect of antihyperlipidemic drug therapy ?
A. Elevated blood pressure
B. * Gastrointestinal disturbance
C. Neurological problems
63
64
65
66
67
68
69
70
71
D. Heart palpitations
E. Migraine headaches
To indicate side effect of Clophellinum (Clonidine)
A. Increasing of arterial pressure
B. Increasing of intraocular pressure
C. * Dryness in the mouth
D. Increasing of glands secretion
E. Increasing of intestine motility
To indicate the antihypertensive drug used for the medical treatment of hypertensive disease with
tachycardia and extrasystoles.
A. Methyldopa
B. Reserpinum
C. * Bisoprolol
D. Clophelinum
E. Dibazolum
What diuretic is used for the removal of hypertensive crisis?
A. * Furosemide
B. Mannitum
C. Triamteren
D. Dichlothiazide
E. Spironolactonum
To indicate the drug – blocker of angiotensin converting enzyme.
A. Anaprilinum
B. * Lisinopril
C. Methyldopa
D. Reserpinum
E. Dibazolum
To indicate– calcium channels blocker hypotensive drug
A. * Nifedepine
B. Captopril
C. Reserpinum
D. Anaprilinum
E. Dibazolum
Mechanism of enalapril action A. Blocks alpha-adrenoreceptors
B. * Blocks carboxypeptidase
C. Blocks M-cholinoreceptors
D. Has diuretic action
E. Has sympatholytic action
Which one of the following drugs decreases de novo cholesterol synthesis by inhibiting the enzyme 3hydroxy-3-methylglutaryl CoA reductase?
A. Clofibrate
B. Niacin
C. Cholestyramine
D. * Lovastatin
E. Gemfibrozil
To indicate diuretic drug that is used for the removal of hypertensive crisis.
A. Oxytocine
B. Dichlothiazide
C. Triamteren
D. Spironolactonum
E. * Furosemide
To indicate what diuretics is used in the complex therapy of hypertensive crisis.
72
73
74
75
76
77
78
79
A. Spironolactonum
B. * Furosemide
C. Manit
D. Dichlothiazide
E. Triamterene
Which is a NOT a common side effect of nitroglycerin?
A. * Blurred vision
B. Flushing
C. Headache
D. Hypotension
E. Hypertension
Calcium channel blockers reduce myocardial oxygen demand by reducing afterload, which is:
A. Blood volume within the heart
B. Pressure within the heart
C. * Pressure against which the heart must pump
D. Contractility of the heart muscle
E. Oxygen demand of the heart
Which of the following nitrate preparations or dosage forms has the longest duration of action?
A. Sublingual nitroglycerin
B. Sublingual isosorbide dinitrate
C. Oral isosorbide dinitrate
D. * Transdermal nitroglycerin patch
E. Validolum
In order to prevent the development of tolerance, the pharmacist instructs the patient to:
A. Apply the nitroglycerin patch every other day
B. Switch to sublingual nitroglycerin when the patient's systolic blood pressure elevates to >140 mm
Hg>
C. * Apply the nitroglycerin patch for 14 hours each and remove for 10 hours at night
D. Use the nitroglycerin patch for acute episodes of angina only
E. Apply the nitroglycerin patch 2 times per day
Binds bile acids in the intestine, thus preventing their return to the liver via the enterohepatic circulation:
A. Niacin
B. * Clofibrate
C. Cholestyramine
D. Probucol
E. Lovastatin
Before administering isosorbide mononitrate, a priority assessment would include:
A. Serum electrolytes
B. Blood urea nitrogen (BUN) and creatinine
C. * Blood pressure
D. Level of consciousness
E. Pressure within the heart
Cause a decrease in liver triacylglycerol synthesis by limiting available free fatty acids needed as
building blocks for this pathway:
A. Cholestyramine
B. * Niacin
C. Clofibrate
D. Probucol
E. Lovastatin
The patient asks how nitroglycerin should be stored while traveling. The pharmacist's best response
would be:
A. "You can protect it from heat by placing the bottle in an ice chest."
B. * "It's best to keep it in its original container away from heat and light."
80
81
82
83
84
85
86
87
C. "You can put a few tablets in a resealable bag and carry in your pant's pocket."
D. "It's best to lock them in the glove compartment of your car to keep them away from heat and light."
E. "Keep the tablets locked in a safe place until you need them."
Patient teaching regarding sublingual nitroglycerin should include which of the following statements:
A. "You can take up to five doses every 3 minutes for chest pain."
B. "Chew the tablet for the quickest effect."
C. "Keep the tablets locked in a safe place until you need them."
D. * "Sit or lie down after you take a nitroglycerin tablet to prevent dizziness."
E. "You can protect it from heat by placing the bottle in an ice chest."
What is the best way to prevent tolerance to nitrates when using the transdermal patches?
A. Leave the old patch on for 2 hours when applying a new patch
B. Apply a new patch off for 24 hours once a week
C. Leave the patch off for 24 hours once a week
D. * Remove the patch at night for 8 hours, and then apply a new patch in the morning
E. Leave the patch off for 12 hours once a week
An annoying side effect of ACE inhibitors that may be minimized by switching to an angiotensin
receptor blocking agent includes:
A. Orthostatic hypotension
B. * A dry, nonproductive cough
C. Fatigue
D. Hypokalemia
E. Hyperkalemia
Which of the following should not be taken concurrently with ACE inhibitors?
A. Lasix
B. Morphine
C. * Potassium
D. Natrium
E. Vicasolum
Inhibits 3-hydroxy-3-methylglutaryl CoA reductase, the rate-limiting step in cholesterol synthesis:
A. * Lovastatin
B. Niacin
C. Cholestyramine
D. Clofibrate
E. Probucol
The doctor would plan to administer which of the following calcium channel blocking agents to a patient
with cerebral arterial spasms following a subarachnoid hemorrhage?
A. amlodipine (Norvasc)
B. diltiazem (Cardizem)
C. * nimodipine (Nimotop)
D. verapamil (Calan)
E. furosemidum
When explaining different medication regimens to treat hypertension during a community education
program, it would be accurate to state that African-Americans probably respond best to which
combination of medications?
A. ACE inhibitors and diuretics.
B. * Diuretics and calcium antagonists
C. Diuretics and beta-blockers
D. ACE inhibitors and beta-blockers
E. All of the above
Labetalol and carvedilol are classified as:
A. Beta-blocking agents
B. Alpha1-blocking agents
C. * Combined alpha- and beta-blocking agents
88
89
90
91
92
93
94
95
96
D. Calcium channel blockers
E. Fibrinolytics
The pharmacist would monitor for reflex tachycardia in a patient receiving which classification of
antihypertensive agents?
A. Calcium channel blockers
B. Cardioselective beta-blockers
C. Nonselective beta-blockers
D. * Direct-acting vasodilators
E. Neuroleptics
ACE inhibitors and angiotensin receptor blockers both work to decrease blood pressure by:
A. Preventing the formation of angiotensin II
B. Enhancing sodium and water resorption
C. Increasing the breakdown of bradykinin
D. * Inhibiting aldosterone secretion
E. All of the above
Which one of the following drugs is useful in treating tachycardia?
A. Phenoxybenzamine
B. Isoproterenol
C. Phentolamine
D. * Propranolol
E. Prazosin
What diuretic has potassium-sparing action?
A. Furosemide
B. Mannitol
C. Dihlothiazid
D. * Triamterene
E. Urea
Equilibrium between plasma and tissue levels for quinidine is reached:
A. * in 1 or 2 days
B. in approximately 1 wk
C. in 1–3 wk
D. in just a few minutes
E. in 2 h
What diuretic is used for forced diuresis?
A. Dihlothiazid
B. Triamterene
C. Spironolactone
D. * Furosemide
E. Diacarb
What diuretic is belonging to osmotic diuretics?
A. Spironolactone
B. * Mannitol
C. Furosemide
D. Dihlothiazid
E. Triamterene
The patient has cranial trauma. The brain edema threat developed in the postoperative period. To
prescribe drug for the removal of this complication.
A. Spironolactone
B. Papaverini hydrochloride
C. * Furosemide
D. Dihlothiazid
E. Diacarb
Equilibrium between plasma and tissue levels for amiodarone is reached:
A. * in 1–3 wk
B. in 1 or 2 d
C. in approximately 1 wk
D. in just a few minutes
E. in 2 h
97 Equilibrium between plasma and tissue levels for IV lidocaine is reached:
A. * in just a few minutes
B. in 1 or 2 d
C. in approximately 1 wk
D. in 1–3 wk
E. in 2 h
98 Drowsiness, paresthesias, muscle twitching, convulsions, changes in mental status (eg, confusion),
hypersensitivity reactions (eg, urticaria, edema, anaphylaxis) – adverse effects of:
A. * Lidocaine
B. Phenytoin
C. Propranolol
D. Disopyramide
E. Verapamil
99 Adverse effects of nitrates are:
A. * hypotension, dizziness, tachycardia
B. bradycardia, bronchospasm, heart failure
C. peripheral edema, headache, heart failure
D. nausea and constipation
E. lightheadedness, weakness, peripheral edema
100
GI problems—nausea, vomiting, flatulence, constipation or diarrhea, abdominal discomfort are the
most common adverse effects of:
A. * dyslipidemic drugs
B. calcium channel blockers
C. beta-adrenergic blocking agents
D. nitrates
E. all of the above
101
Which of the following adverse effects is associated with nitroglycerin?
A. Bronchospasm
B. * Throbbing headache
C. Bradycardia
D. Sexual dysfunction.
E. Anemia.
102
All of the following statements concerning nitroglycerin are correct EXCEPT:
A. It causes an elevation of intracellular cGMP.
B. It undergoes significant first-pass metabolism in the liver.
C. It may cause significant reflex tachycardia.
D. * It can not cause throbbing headache.
E. It can cause postural hypotension.
103
Which of the following adverse effects is associated with nitroglycerin?
A. Hypertension
B. * Increasing of intracranial pressure
C. Bradycardia
D. Sexual dysfunction.
E. Anemia.
104
Nitroderm is used:
A. Antiarrhythmic action is via (-adrenergic blockade
B. Ventricular premature depolarizations
C. * Angina pectoris prophylaxis
D. Conversion of atrial fibrillation to sinus rhythm
E. Treatment of ventricular fibrillation
105
Which one of the following is a specific alpha-adrenergic antagonist:
A. Atenolol
B. Timolol
C. Labetalol
D. * Terazosin
E. Sotalol
106
Agent, that inhibits 3-hydroxy-3-methylglutaryl CoA reductase, the rate-limiting step in cholesterol
synthesis:
A. * Pravastatin
B. Niacin
C. Cholestyramine
D. Clofibrate
E. Probucol
107
Adverse effects of organic nitrates are:
A. nausea and constipation
B. bradycardia, bronchospasm, heart failure
C. peripheral edema, headache, heart failure
D. * throbbing headache, dizziness, tachycardia
E. lightheadedness, weakness, peripheral edema
108
What diuretic is used for the removal of hypertensive crisis?
A. Dichlothiazide
B. * Furosemide
C. Spironolactonum
D. Mannitum
E. Triamteren
109
Indicate the antiarrytmic drug from of adrenoblockers group
A. Nitroglycerin
B. Amiodaron
C. * Metoprolol
D. Verapamil
E. Asparcam
110
The drug related to the group of calcium antagonists
A. Apressinum
B. Prazosinum
C. * Amlodipine
D. Reserpine
E. Papaverini hydrochloridum
111
Factors causing a susceptibility to urinary tract infect include:
A. urinary tract obstruction
B. diabetes mellitus
C. pregnancy
D. * All of the above
E. None of the above
112
In the nephrotic syndrome:
A. the prognosis is better in males than in female patients
B. * a generalized edema is present, favourable treatment includes the management of any underlying
disease
C. an intermittent microscopic hematuria suggests advanced parenchymal damage
D. the administration of steroids is always ineffective
E. All of the above
113
Mechanism of action of cardiac glycosides
A.
B.
C.
D.
E.
Block the activity of COX2
Cause the dilatation of coronary vessels
Block the beta-adrenoreceptors
Block the calcium channels
* Block the activity of adenosine-tryphosphatase (ATP-ase)
114
Dose of which of the following may not be altered with deranged value of GFR?
A. * Cefipime
B. Cefoperazone
C. Cefuroxime
D. Tetracycline
E. All of the above
115
A drug that can selectively supress automaticity in Purkinje fibers compared to the sinus node:
A. verapamil
B. atenolol
C. diltiazem
D. propranolol
E. * lidocaine
116
Carbonic anhydrase inhibitor
A. furosemide (Lasix)
B. bumetanide (Bumex)
C. * acetazolamide (Diamox)
D. chlorothiazide (Diuril)
E. spironolactone (Aldactone)
117
About 35% of the filtered load of sodium chloride is reabsorbed here:
A. thin limb -- --loop of Henle
B. collecting duct
C. distal convoluted tubule
D. * thick ascending limb -- loop of Henle
E. cortical collecting tubule
118
A drug, or class of drugs, that has been documented to reduce mortality following a myocardial
infarction:
A. Class Ia antiarrhythmics
B. Class Ic antiarrhythmics
C. * Class II antiarrhythmics
D. Class III antiarrhythmics
E. digoxin
119
Mechanism of action: spironolactone (Aldactone)
A. through osmotic effects
B. through enzyme inhibition
C. * through interaction with hormonal receptors
D. all of the above
E. none of the above
120
GI upset, tinnitus and cinchonism are common side effects of:
A. lidocaine
B. digoxin
C. propranolol
D. * quinidine
E. amiodarone
121
Diuretics acting on specific membrane transport proteins:
A. loop diuretics
B. thiazides
C. amiloride (Midamor)
D. triamterene (Dyrenium)
E. * all of the above
122
An antiarrhythmic drug with the most "non-selective" mechanism of action:
A. lidocaine
B. Quinidine
C. Propranolol
D. Sotalol
E. * amiodarone
123
Diuretic mechanism(s):
A. through osmotic effects (preventing water reabsorption)
B. enzyme inhibition
C. interaction with hormonal receptors
D. * all of the above
E. none of the above
124
Loop diuretics -- effects on ionic excretion:
A. * increased sodium excretion, increased calcium loss
B. decreased magnesium loss
C. decreased potassium loss
D. all of the above
E. none of the above
125
A primary mechanism by which Class I antiarrhythmic drugs produce their therapeutic effect in
patients with ventricular arrhythmias:
A. Block L-type Ca channels
B. Block beta-adrenergic receptors
C. * Increase the Effective Refractory Period
D. Increase vagal tone to the AV node
E. Increase the dispersion of refractoriness
126
Primary indications for treatment of cardiac arrhythmias include all of the following EXCEPT:
A. * Arrhythmias that reduce cardiac output
B. Arrhythmias that are prone to degenerate into more serious arrhythmias
C. Arrhythmias that cause vascular stasis
D. Arrhythmias that increase the risk of stroke
E. Monomorphic premature ventricular beats
127
A side effect of furosemide that can increase the automaticity of ectopic cardiac pacemakers:
A. hypercalcemia
B. hyperkalemia
C. * hypokalemia
D. hyponatremia
E. hypothyroidism
128
In the treatment of exertional angina:
A. * nitroglycerin reduces O2 demand by decreasing preload
B. nitroglycerin increases O2 supply by dilating atherosclerotic vessels
C. attacks frequently occur during sleep
D. nitroglycerin increases cardiac myocyte cAMP levels
E. beta blocking agents have no beneficial action
129
Diltiazem:
A. * decreases calcium entry through L type channels
B. increases total peripheral resistance
C. increases cardiac output
D. increases heart rate
E. increases gastrointestinal motility
130
A drug effect that is produced by therapeutic doses of both propranolol and amiodarone is blockade
of:
A. cardiac Na channels
B.
C.
D.
E.
cardiac K channels
* beta-adrenoceptors
alpha-adrenoceptors
L-type Ca channels
131
The most common cellular mechanism responsible for production of cardiac arrhythmias in patients
with ischemic heart disease:
A. * Hypokalemia
B. depressed automaticity
C. sick sinus syndrome
D. reentrant excitation
E. hormonal imbalance
132
Mechanism of action: acetazolamide (Diamox)
A. through osmotic effects
B. * through enzyme inhibition
C. interacting with hormonal receptors
D. all of the above
E. none of the above
133
In a patient with a high risk for coronary artery disease (LDL cholesterol 200 mg/dL, normal
tryglycerides), the best lipid lowering drug would be:
A. nicotinic acid
B. gemfibrozil
C. atorvastatin
D. colestipol
E. * cholestyramine
134
Pharmacological blockade of sodium and chloride cotransport at the distal convoluted tubule:
A. bumetanide (Bumex)
B. furosemide (Lasix)
C. * chlorothiazide (Diuril)
D. triamterene (Dyrenium)
E. none of the above
135
What diuretic has potassium-sparing action?
A. Urea
B. Furosemide
C. Mannitol
D. Dihlothiazid
E. * Triamterene
136
Indicate the antiarrythmic drug which exerts membrane-stabilizing action.
A. Atenolol
B. * Quinidine sulfate
C. Nitroglycerin
D. Digoxin
E. Isadrinum
137
What diuretic is used for forced diuresis?
A. Dihlothiazid
B. Triamterene
C. Spironolactone
D. * Furosemide
E. Diacarb
138
Nitrates relieve angina pain by reducing preload, which is:
A. * Oxygen demand of the heart
B. Pressure within the heart
C. Blood volume within the heart
D. Pressure against which the heart must pump
E. Contractility of the heart muscle
139
What diuretic is belonging to thiazide diuretics?
A. Spironolactone
B. Mannitol
C. Furosemide
D. * Dihlothiazid
E. Triamterene
140
Acetazolamide (Diamox)
A. inhibits carbonic anhydrase
B. may produce metabolic acidosis
C. increases potassium secretion
D. * all of the above
E. none of the above
141
Indicate the Verapamil possible side effect, as antiarrhythmic drug
A. Tachycardia
B. Bronchospasm
C. Stenocardia
D. Mouth dryness
E. * Decrease of artery pressure (hypotension)
142
Osmotic diuretic is
A. furosemide (Lasix)
B. bumetanide (Bumex)
C. * mannitol (Osmitrol)
D. thiazides
E. None of the above
143
Nifedipine and verapamil both
A. * cause coronary vasodilation
B. depress atrioventricular nodal conduction
C. cause reflex tachycardia
D. stimulate calcium entry into cells
144
Primary site of action of loop diuretics:
A. distal tubule
B. collecting duct
C. thin descending limb of the loop of Henle
D. * thick ascending limb of the loop of Henle
E. at the site of glomerular filtration
145
Loop diuretics are:
A. * rapidly absorbed
B. eliminated by renal secretion only
C. eliminated by glomerular filtration only
D. loop diuretics act on the interstitial side of the principal cell
E. indomethacin (Indocin) increases loop diuretic clearance
146
Which of the following agents is selective for beta1-adrenergic receptor?
A. Metaproterenol
B. Propranolol
C. * Metoprolol
D. Terbutalin
E. Timolol
147
Loop diuretics: Effects on magnesium and calcium excretion A. increase in magnesium secretion
B. increase in calcium secretion
C. hypomagnesemia in patients (prolonged loop diuretic use)
D. No hypocalcemia in patients (prolonged loop diuretic use)
E. * all of the above
148
Selective blockade of beta1-adrenergic receptors:
A. Labetalol
B. Prazosin
C. Timolol
D. * Atenolol
E. Sotalol
149
Furosemide (Lasix) toxicity:
A. * ototoxicity
B. hypouricemia
C. hypermagnesemia
D. severe fluid overload
E. hypernatremia
150
Agent is a specific alpha-adrenergic antagonist:
A. * Prazosin
B. Timolol
C. Labetalol
D. Atenolol
E. Sotalol
151
Indicate the peculiarility of Furosemide action
A. Slow effect development
B. * Fast effect development
C. The duration of action (10-14 days)
D. Low diuretic activity
E. It doesn’t influence K concentration in the blood
152
Indicate the diuretic of immediate diuretic action influencing the ascending part of Henle loop:
A. Euphyllinum
B. Spironolactonum
C. * Furosemidum
D. Triamterenum
E. Diacarbum
153
Labetalol is:
A. * a selective alpha-adrenergic antagonist as well as a nonspecific beta-adrenergic antagonist
B. is a specific alpha-adrenergic antagonist
C. is a nonspecific beta-adrenergic antagonist
D. is a selective beta1-adrenergic blocker
E. Neither
154
To indicate marching 8-oxychinoline for the medical treatment of urinary infections.
A. Rifampine
B. * Nitroxoline
C. Biseptol
D. Ampicilline
E. Furosemide
155
Which of the following agents is pure beta1-adrenergic agonists?
A. Albuterol
B. Propranolol
C. * Bisoprolol
D. Isoproterenol
E. Timolol
156
To indicate quinolone derivated , which is used mainly at infections of urinary tract.
A. * Nalidixic Acid
B. Biseptol
C. Furosemide
D. Aspirine
E. All of the above
157
Weakness or dizziness, especially with activity or exercise – adverse effects of:
A. * Propranolol
B. Phenytoin
C. Lidocaine
D. Disopyramide
E. Verapamil
158
Adverse effects of beta-adrenergic blocking agents are:
A. * bradycardia, bronchospasm, heart failure
B. hypotension, dizziness, lightheadedness
C. headache, heart failure, pulmonary edema,
D. nausea and constipation
E. hypotension, dizziness, tachycardia
159
Adverse effects of calcium channel blockers are:
A. * dizziness, lightheadedness, peripheral edema, constipation
B. bradycardia, bronchospasm, heart failure
C. hypotension, tachycardia, headache
D. palpitations and headache
E. hypotension, dizziness
160
Calcium channel blockers with prolong action:
A. Nifedipine
B. Verapamil
C. Diltiazem
D. * Amlodipine
E. Nebivolol
161
Which of the following agents is selective for beta1-adrenergic receptor?
A. Metaproterenol
B. Propranolol
C. * Nebivolol
D. Terbutalin
E. Esmolol
162
Call calcium channel blockers with prolong action:
A. * Felodipine
B. Verapamil
C. Diltiazem
D. Nifedipine
E. Nebivolol
163
Amlodipine and verapamil both
A. * cause coronary vasodilation
B. depress atrioventricular nodal conduction
C. cause reflex tachycardia
D. stimulate calcium entry into cells
164
Which of the following agents is selective for beta1-adrenergic receptor?
A. Salmeterol
B. Propranolol
C. * Bisoprolol
D. Terbutalin
E. Timolol
165
Which one of the following drugs is useful in treating tachycardia?
A. * Bisoprolol
B. Isoproterenol
C. Phentolamine
D. Phenoxybenzamine
E. Prazosin
166
Weight gain & edema occur in patients with congestive heart failure in response to:
A. * increased aldosterone levels
B. increased diuresis
C. decreased renin levels decreased total peripheral resistance
D. decreased venous capacitance
167
The mechanism by which digoxin causes a positive inotropic effect on heart tissue is by:
A. increasing ATP hydrolysis
B. stimulating the Na/K pump
C. * increasing intracellular Na and Ca
D. sensitizing the heart to catecholamines
E. inhibiting the Na/H exchanger
168
A drug that is NOT known to increase the effect or toxicity of digoxin when given concominantly:
A. furosemide
B. verapamil
C. amiodarone
D. * propranolol
E. quinidine
169
A drug used in the treatment of systolic congestive heart failure and produces its effects by inhibiting
the cardiac Na/K pump:
A. Furosemide
B. Nifedipine
C. captopril
D. clonidine
E. * digoxin
170
In the treatment of congestive heart failure, choose a drug that would reduce dyspnea and pulmonary
edema by decreasing the left ventricular preload:
A. Hydralazine
B. propranolol
C. * isosorbide dinitrate
D. Dobutamine
E. digoxin
171
Sodium nitroprusside:
A. * is used to lower blood pressure in a hypertensive emergency
B. increases smooth muscle cAMP levels
C. has a slow onset and long duration of action
D. is given orally
E. inhibits nitric oxide formation
172
To indicate the antihypertensive drug used for the medical treatment of hypertensive disease with
tachycardia and extrasystoles.
A. Reserpinum
B. Methyldopa
C. * Bisoprolol
D. Clophelinum
E. Dibazolum
173
To indicate the possible side effect of propranolol?
A. * Bronchial spasm
B. Tachycardia
C. Ortostatic collapse
D. Rise of АP
E. Development of tolerance
174
To indicate an antihypertensive drug from the calcium channels blockers.
A.
B.
C.
D.
E.
Anaprilinum
Captopril
Dibazolum
* Nifedipine
Reserpinum
175
Acebutolol is
A. * Beta-adrenoreceptor antagonist
B. Calcium channel blocker
C. Beta-adrenoreceptor agonist
D. Angiotensin II receptor antagonists
E. ACE inhibitor
176
The patient suffer from hypertensive disease with bradiarrhythmia. What drug is necessary to
prescribe?
A. Papaverinum
B. Platyphyllini hydrotartras
C. Ceftriaxone
D. * Methyldopa
E. Paracetamol
177
Indicate the Verapamil possible side effect, as antiarrhythmic drug
A. Tachycardia
B. Bronchospasm
C. Stenocardia
D. Mouth dryness
E. * Decrease of artery pressure (hypotension)
178
What diuretic has potassium-sparing action?
A. Furosemide
B. Mannitol
C. Dihlothiazid
D. * Triamterene
E. Urea
179
What diuretic is used for forced diuresis?
A. Dihlothiazid
B. Triamterene
C. Spironolactone
D. * Torasemide
E. Diacarb
180
Treatment of digoxin overdose may include all the following EXCEPT
A. withdrawal of digoxin
B. intravenous injection of Fab fragments of digitalis antibodies
C. potassium supplements
D. antiarrhythmic drugs
E. * cholestyramine to break enterohepatic circulation
181
The administration of digoxin to a patient with congestive heart failure results eventually in a
decrease in all of the following parameters EXCEPT
A. heart rate
B. end-diastolic pressure
C. arteriolar resistance
D. * AV node refractoriness
E. renal sodium and water retention
182
Each of the following agents is useful to treat diastolic heart failure EXCEPT:
A. lisinopril
B. * digoxin
C. verapamil
D. furosemide
E. carvedilol
183
Which of the following maneuvers is the most likely to improve renal perfusion in a patient treated
for acute cardiac failure?
A. administration of phenylephrine
B. administration of propranolol
C. injection of esmolol
D. * administration of dopamine
E. administration of epinephrine
184
The drugs whose international common name ends in - sentan such as bosentan, are:
A. ACE (angiotensin converting enzyme) inhibitors
B. progestogens
C. * antagonists of endothelin receptors
D. agonists of adenosine receptors
E. directly acting cholinomimetic drugs
185
Clonidine lowers blood pressure:
A. by inhibiting alpha-1 adrenergic receptors
B. by inhibiting angiotensin converting enzyme
C. * by stimulating central alpha-2 adrenergic receptors
D. by inhibiting endothelin receptors
E. by inhibiting beta-adrenergic receptors
186
Sartan: the drugs whose international common name ends by the suffix - sartan are:
A. antineoplastic agents
B. antiemetic agents
C. antihypertensive drugs
D. * angiotensin II receptor antagonists
E. enkephalin receptor antagonists
187
Pril: the drugs whose international common name ends in the suffix – pril are:
A. anorectics
B. inhibitors of type V phosphodiesterases
C. * ACE (angiotensin converting enzyme) inhibitors
D. agonists of alpha- adrenergic receptors
E. antidiarrheal drugs
188
During treatment, a patient develops a drug-induced dry cough. The drugs possibly responsible for
this cough are:
A. codeine
B. diclofenac
C. * perindopril
D. Acetaminophen
E. All of the above
189
A side effect of furosemide that can increase the automaticity of ectopic cardiac pacemakers:
A. hypercalcemia
B. hyperkalemia
C. * hypokalemia
D. hyponatremia
E. hypothyroidism
190
Antihypertensive drug belonging to the same class:
A. Doxazosin (Cardura), prazosin (Minipress), metoprolol (Lopressor)
B. * nifedipine (Procardia, Adalat), verapamil (Isoptin, Calan), diltiazem (Cardiazem)
C. Clonidine (Catapres), guanabenz (Wytensin),terazosin (Hytrin)
D. Lisinopril (Prinvivil, Zestril), fosinopril (Monopril), guanadrel (Hylorel)
E. None of the above
191
Mechanism of action: prasosin (Minipress)
A.
B.
C.
D.
E.
* alpha-1 receptor blocker
beta receptor blocker
phosphodiesterase inhibitor
calcium channel blocker
none of the above
192
Side effects of this antihypertensive agent includes tachycardia, angina, reversible lupus-like
syndrome
A. propranolol (Inderal)
B. mecamylamine (Inversine)
C. * hydralazine (Apresoline)
D. Diazoxide (hyperstat)
193
Vasoconstriction, aldosterone secretion, and renin release suppression occur upon activation of the
renin-angiotensin-aldosterone system. How would captopril (Capoten) affect these responses?
A. * blocks all three
B. blocks only vasoconstriction
C. blocks all except vasoconstriction
D. no effect
E. Block renin release
194
Antihypertensive drug LEAST likely to elevate serum lipids:
A. propranolol (Inderal)
B. metoprolol (Lopressor)
C. chlorothiazide (Diuril)
D. * diltiazem (Cardiazem)
E. Both chlorothiazide and propranolol
195
Mechanism of action: prazosin (Minipress)
A. * alpha-1 receptor blocker
B. beta receptor blocker
C. phosphodiesterase inhibitor
D. calcium channel blocker
E. none of the above
196
Mechanism of action: diltiazem (Cardiazem)
A. phosphodiesterase inhibitor
B. * blockade of calcium channels
C. alpha-1 receptor antagonists
D. beta-1 receptor antagonist
E. all of the above
197
I.v. route of administration; few side effects; effective in treating hypertensive crisis:
A. nifedipine (Procardia, Adalat)
B. * nitroprusside sodium (Nipride)
C. nicardipine (Cardene)
D. all of the above
E. none of the above
198
Total peripheral resistance (TPR) is a determining factor for mean arterial pressure, i.e. Mean arterial
pressure (MAP) = cardiac output x total peripheral resistance (TPR). What are correct relationships
between norepinephrine, minoxidil, and lisinopril and TPR?
A. minoxidil: TPR increases
B. fosinopril: TPR increases
C. * norepinephrine: TPR increases
D. phenoxybenzamine: TPR increases
E. None of the above
199
Ganglionic blocker
A. prazosin (Minipress)
B. hydralazine (Apresoline)
C. * mecamylamine (Inversine)
D. nicardipine (Cardene)
E. None of the above
200
Minoxidil (Loniten) administration produces arteriolar vasodilation with little effect on capacitance
vessels. Drug(s) acting similarly:
A. nitroprusside sodium (Nipride)
B. propranolol (Inderal)
C. * hydralazine (Apresoline)
D. all of the above
E. None of the above
201
Blocks both alpha and beta receptors:
A. timolol (Blocadren)
B. * labetalol (Trandate, Normodyne)
C. propranolol (Inderal)
D. diazoxide (Hyperstat)
E. all of the above
202
Hypertensive emergencies: indicated using of
A. diltiazem (Cardiazem)
B. * nitroprusside sodium (Nipride)
C. reserpine
D. phenylephrine (Neo-Synephrine)
E. all of the above
203
ACE inhibitor is
A. nifedipine (Procardia, Adalat)
B. * captopril (Capoten)
C. timolol (Blocadren)
D. methyldopa (Aldomet)
E. None of the above
204
Antihypertensive: action based on inhibition of norepinephrine release from adrenergic nerve
endings:
A. propranolol (Inderal)
B. * guanadrel (Hylorel)
C. mecamylamine (Inversine)
D. phentolamine (Regitine)
E. None of the above
205
Causes of hypertension:
A. Cushing's disease
B. oral contraceptives
C. acromegaly
D. polycystic kidney disease
E. * all of the above
206
Starling's Law and related matters:
A. As blood volume returning to the hard increases,ventricular dilation is reduced
B. Generally, increase preloaded an increase contractility lead to decreased stroke volume
C. * According to Starling's Law, increased ventricular stretch usually leads to increased myocardial
contractility
D. all of the above
E. None of the above
207
The most specific agent for prevention of asthma is:
A. * Salbutamolum
B. Libexinum
C. Adrenalini hydrochloridum
D. Pertussinum
E. Mucaltinum
208
Drugs that may decrease theophylline clearance include:
A. * Cimetidine, ciprofloxacin
B. ranitidine
C. phenytoin
D. None of the above
E. All of the above
209
Theophylline clearance may be reduced by:
A. phenobarbital
B. warfarin
C. tobacco smoking
D. phenytoin
E. * ciprofloxacin
210
Factors which are important to consider in selecting a maintenance dose of theophylline in a patient
include all of the following EXCEPT:
A. smoking history
B. concomitant medication administration
C. presence of congestive heart failure or cor pulmonale
D. presence of hepatic failure
E. * presence of renal insufficiency
211
Manifestations of theophylline toxicity may include all of the following EXCEPT:
A. tremors
B. seizures
C. tachyarrhythmias
D. vomiting
E. * hyperkalemia
212
In the case of an oral overdose of theophylline, clearance may be enhanced by: induction of emesis
A. * oral pulse dose charcoal
B. administration of cimetidine
C. lavage of nasogastric contents
D. forced diuresis
E. None of the above
213
Regarding its actions, cromolyn is best described as:
A. bronchodilator
B. anticholinergic
C. beta agonist
D. * inhibitor of mast cell degranulation
E. glucocorticoid
214
Beta-2 selective adrenergic agonists include all EXCEPT:
A. terbutaline
B. albuterol
C. metaproterenol
D. * isoproterenol
E. pirbuterol
215
Adverse effects of beta-2 adrenergic bronchodilators include all of the following EXCEPT:
A. nervousness
B. headache
C. tachycardia
D. tremulousness
E. * lethargy
216
Which of the following work through cholinergic receptor antagonism?
A. isoetharine
B. cromolyn
C. ephedrine
D. * ipratropium
E. salmeterol
217
Which of the following pharmacologic agents antagonize adenosine?
A. * theophylline
B. glucocorticoids
C. cromolyn sodium
D. propranolol
E. terbutaline
218
Theophylline, ethacrynic acid, and furosemide:
A. interfere with active renal tubular reabsorption of sodium
B. * increase urine volume
C. are not effective in the treatment of congestive heart failure
D. decrease excretion of sodium in the urine
E. may precipitate in renal tubules and cause acute angiospastic necrosis
219
The agent with immune boosting activity yet recently implicated with causing severe depression and
suicidal ideation is
A. echinacea
B. * interferon
C. dronabinol (Marinol)
D. megestrol (Megace)
E. all of the above
220
Examples of autoimmune diseases
A. rheumatoid arthritis
B. insulin-dependent diabetes mellitus
C. systemic lupus erythematosis
D. rheumatoid arthritis & systemic lupus erythematosis
E. * all of the above
221
Which of the following asthma severity categories has a treatment including inhaled low-dose
steroid, cromolyn, nedocromil, zafirlukast or zileuton?
A. Mild intermittent asthma
B. * Mild persistent asthma
C. Moderated persistent asthma
D. Severe persistent asthma
E. Exercise-induced asthma
222
Which of the following drugs acts through a cytosolic receptor?
A. Cromolyn sodium
B. * Beclomethasone
C. Zafirlukast
D. Omalizumab
E. None of the above
223
Which of the following corticosteroids is used for quick relief from asthma?
A. Triamcinolone
B. * Dexamethasone
C. Budesonide
D. Beclomethasone
E. Fluticasone
224
Which of the following is not true about Salmeterol?
A. administered by inhalation
B. long acting anti-asthmatic with a duration of 12 hour
C. * can lead to transient increase in PaO2, especially in poorly ventilated lung tissue
D. associated with improvement in patients homozygous for glycine at the B-16 locus of the betareceptor
E. none of the above
225
Which of the following is not true about Ipratropium bromide:
A. Anticholinergic agent administered by inhalation
B. better bronchodilator than Atropine
C. * better anti-asthmatic than Epinephrine
D. has no effect on mucocilliary clearance
E. none of the above
226
Which of the following is not a quick relief medication for asthma?
A. Epinephrine
B. Ipratropium bromide
C. Prednisolone
D. * Flunisolide
E. All of the above
227
The following is not true of beta-2 adrenergic agonists:
A. cause bronchodilation by decreasing cytosolic Ca2+
B. cause hypotension
C. * decrease mucocilliary activity
D. decrease bronchial vascular permeability
E. increase heart rate
228
Which of the following can be administered orally and by inhalation?
A. Epinephrine
B. Albuterol
C. Pirbuterol
D. * Terbutaline
E. Isoproterenol
229
Which of the following drug combinations would produce a synergistic effect
A. Amoxicillin+ ampicillin
B. Cephalexin+ penicillin V
C. Sulfamethoxazole+ trimethoprim
D. Vancomycin+ gentamicin
E. * Sulfamethoxazole+ trimethoprim and Vancomycin+ gentamicin
230
The drug of choice for methicillin-resistant Staphylococcus aureus is:
A. oxacillin
B. gentamicin
C. * vancomycin
D. piperacillin
E. None of the above
231
Tetracyclines:
A. are bacteriostatic in vitro
B. are bactericidal in vitro
C. are effective against rickettsiae
D. * are bacteriostatic in vitro, are effective against rickettsiae
E. interfere primarily with cell wall synthesis
232
The drug of choice for Streptococcus pneumonia (pneumococcus) in a patient with no drug allergies
is:
A. erythromycin
B. * penicillin
C. vancomycin
D. ceftriaxone
E. gentamicin
233
Aminoglycosides are:
A. excreted renally
B. bactericidal
C. potentially toxic to the auditory and vestibular portions of the eighth cranial nerve
D. potentially nephrotoxic
E. * All of the above
234
The route by which isoniazid is usually administered is:
A. * oral
B. intramuscular injection of solution
C. intramuscular injection of suspension
D. subcutaneous injection of solution
E. intravenous
235
Rifampin is used mainly in the treatment of:
A. cholera
B. typhoid fever
C. * tuberculosis
D. rickettsial diseases
E. pseudomonas infections
236
Impaired vision is an adverse effect of:
A. carbenicillin
B. * ethambutol
C. rifampin
D. colistin
E. cycloserine
237
Various drugs may induce vitamin deficient states as an undesirable side effect. Vitam B6
(pyridoxine) deficiency may be related to taking:
A. estrogen-containing oral contraceptives
B. colchicine
C. * isoniazid
D. All of the above
E. None of the above
238
Most adults who are at high risk of developing pulmonary tuberculosis should be treated with:
A. isoniazid 100 mg/day for 6 months
B. * isoniazid 300 mg/day for 12 months
C. ethambutol 400 mg/day for 12 months
D. rifampin 600 mg/day for 24 months
E. No treatment is necessary
239
An antitubercular agent which is associated with the development of ocular toxicity is:
A. rifampin
B. * ethambutol
C. isoniazid
D. streptomycin
E. para-aminosalicylic acid
240
Which of the following antitubercular agents is associated with the development of ototoxicity?
A. rifampin
B. ethambutol
C. isoniazid
D. * streptomycin
E. para-aminosalicylic acid
241
Which of the following antibiotics is most closely associated with the development of ocular
toxicity?
A. kanamycin
B. penicillin G
C. tetracycline
D. isoniazid
E. * ethambutol
242
Which of the following antibiotics is most closely associated with the development of hepatitis?
A. kanamycin
B. penicillin G
C. tetracycline
D. * isoniazid
E. ethambutol
243
Which of the following antibiotics is most closely associated with the development of renal and
ototoxicity?
A. * kanamycin
B. penicillin G
C. tetracycline
D. isoniazid
E. ethambutol
244
Which of the following agents used in the treatment of pulmonary tuberculosis has the side effect of
loss of perception of the color green?
A. isoniazid
B. rifampin
C. * ethambutol
D. streptomycin
E. para-amino salicylic acid
245
Isoniazid-induced liver damage:
A. occurs primarily in patients under 30 years of age
B. occurs with increased frequency in patients receiving concomitant ethambutol therapy
C. * is probably due to the formation of a toxic hydrazine metabolite that binds to liver protein
D. is frequently associated with allergic manifestations such as eosinophilia, fever, and rash
E. All of the above
246
Which of the following agents used in the treatment of pulmonary tuberculosis has a toxicity of
producing a lupus-like syndrome?
A. * isoniazid
B. rifampin
C. ethambutol
D. streptomycin
E. para-amino salicylic acid
247
Tubercle bacilli exist in tuberculous patients in three pools extracellular, intracellular, and necrotic
caseum. The only drug which is bactericidal for all three is:
A. isoniazid
B. ethambutol
C. pyrazinamide
D. * rifampin
E. streptomycin
248
All of the following are considered primary drugs in the current therapeutic approaches to the
treatment of tuberculosis EXCEPT:
A. streptomycin
B. * para-aminosalicylic acid
C. isoniazid
D. rifampin
E. ethambutol
249
Which of the following antitubercular drugs has hyperuricemia as one of its well-documented
untoward effects?
A. streptomycin
B. isoniazid
C. cycloserine
D. rifampin
E. * pyrazinamide
250
Which of the following is a bactericidal antitubercular drug that has an inhibitory action on cell wall
synthesis?
A. * isoniazid
B. rifampin
C. ethambutol
D. streptomycin
E. para-aminosalicylic acid
251
Ethambutol has become a primary treatment drug in the therapy of tuberculosis because of its
infrequent side effects. The only significant toxic effect is:
A. bone marrow suppression
B. renal tubular damage
C. * retrobulbar neuritis
D. hepatitis
E. Stevens-Johnson Syndrome
252
Rifampicine is used mainly in the treatment of :
A. Cholera
B. Typhoid fever
C. * Tuberculosis
D. Rickettsial diseases
E. Pseudomonas infections
253
Patients having a history of a severe, immediate reaction to penicillin:
A. may be given a cephalosporin without concern
B. have a definite risk of reaction to any cephalosporin
C. have a low risk of having a reaction to a broad spectrum antipseudomonal penicillin
D. have a high risk of hypersensitivity to a broad spectrum antipseudomonal penicillin
E. * have a definite risk of reaction to any cephalosporin & have a high risk of hypersensitivity to a
broad spectrum antipseudomonal penicillin
254
What agents are antiviral:
A. Interferonum
B. Biochinolum
C. Oxolinum
D. Remantadinum
E. * Interferonum, Oxolinum & Remantadinum
255
Choose the necessary component of therapy of pneumonia complicated by pleurisy with effusion:
A. Inhibitors of proteinases;
B. Diuretic;
C. * Antibiotics;
D. Antiviral therapy;
E. Cardiac glycosides.
256
Most common fungal infection in HIV patients
A. Cryptococcosis
B. * candidiasis
C. histoplasmosis
D. Neither
E. All of the above
257
Agents useful in treating mycobacterial infections in HIV patients:
A. clarithromycin (Biaxin)
B. ethambutol (Myambutol)
C. amikacin (Amikin)
D. * all of the above may be useful
E. None of the above
258
Properties of tetracycline (Achromycin):
A.
B.
C.
D.
E.
259
A.
B.
C.
D.
E.
260
A.
B.
C.
D.
E.
261
A.
B.
C.
D.
E.
262
A.
B.
C.
D.
E.
263
A.
B.
C.
D.
E.
264
A.
B.
C.
D.
E.
265
A.
B.
C.
D.
E.
266
A.
B.
C.
inhibitor of bacterial cell wall synthesis
* drug of choice in treating typhus
cleared primarily by the liver
All of the above
None of the above
Suitable for treatment of bacterial meningitis caused by H. Influenzae:
* cefotaxime (Claforan)
cephalexin (Keflex)
cephalothin (Keflin)
cefadroxil (Duricef, Ultracef)
None of the above
Antibiotic's clearance from the body most likely influenced by severe hepatic disease:
Penicillins
clindamycin (Cleocin)
rifampin (Rimactane)
* clindamycin & rifampin
None of the above
Associated with "gray baby syndrome" which is characterized by pallor, cyanosis, and even death.
Tetracyclines
* chloramphenicol (Chloromycetin)
third-generation cephalosporins
Tetracyclines & chloramphenicol
None of the above
Special concerns in using antimicrobials in newborns and young children:
Sulfonamides can be safely given to newborns
Tetracyclines should not be administered to children betlow the age of eight
Newborns should not be given chloramphenical because they are unable to metabolize the drug
adequately.
* Tetracyclines should not be administered to children betlow the age of eight & Newborns should
not be given chloramphenical because they are unable to metabolize the drug adequately
None of the above
Antibacterials regarded as generally safe to prescribe in pregnancy.
Erythromycins
Cephalosporins
Tetracyclines
* Erythromycins & Cephalosporins
All of the above
Ototoxicity and nephrotoxicity associated with this antibiotic:
cefotaxime (Claforan)
* amikacin (Amikin)
aztreonam (Azactan)
ceftriaxone (Rocephin)
cefotaxime & amikacin
Aminoglycosides antibacterial is
Hetacillin
Aztreonam
* gentamicin (Garamycin)
vancomycin (Vancocin)
gentamicin & aztreonam
FALSE statement about penicillin G
treatment of choice for viridans group streptococcal endocarditis.
Pen G and Pen V are the two natural penicillins.
Pen G can be combined with procaine, extending drug half-life
D. * Pen G most effective when given orally
E. All of the above
267
Inhibits bacterial cell wall synthesis:
A. Streptomycin
B. * vancomycin (Vancocin)
C. doxycycline (Vibramycin, Doryx)
D. gentamicin (Garamycin)
E. None of the above
268
Mechanism of action: amoxicillin (Amoxil)
A. inhibits bacterial cell wal biosynthesis
B. interferes with transpeptidation preventing peptidoglycan chain crosslinking.
C. Inhibits protein synthesis
D. * inhibits bacterial cell wal biosynthesis & interferes with transpeptidation preventing peptidoglycan
chain crosslinking
E. interferes with transpeptidation preventing peptidoglycan chain crosslinking & Inhibits protein
synthesis
269
Combination of metronidazole (Flagyl) and penicillin in treating an abscess caused by beta-lactamase
producing Bacteriodes and anaerobic streptococci is an example of:
A. synergistic drug treatment
B. antagonistic drug effects
C. * additive drug effects
D. none of the above
E. synergistic drug treatment & additive drug effects
270
Beta-lactamase inhibitor:
A. vancomycin (Vancocin)
B. sulbactam
C. clavulanate
D. * sulbactam & clavulanate
E. None of the above
271
Antifungal polyene macrolide that preferentially binds to fungal ergosterol which alters cellular
permeability.
A. Ketoconazole
B. * amphotericin B
C. Flucytosine
D. Grisefulvin
E. Clotrimazole
272
An azole most commonly used for topical treatment of candidiasis:
A. amphotericin B
B. * clotrimazole
C. Griseofulvin
D. Flucytosine
E. none of the above
273
Polyene macrolide only used topically, due to systemic toxicity; active against most Candida species-most commonly used for suppression of local candidal infection.
A. Miconazole
B. Terbinafine
C. * Nystatin
D. Fluconazole
E. Flucytosine
274
Inhibition of fungal squalene epoxidase is this antifungal drug's mechanism of action:
A. Ketoconazole
B. Fluconazole
C. * Terbinafine
D. Nystatin
E. Ketoconazole & Fluconazole
275
Most potent of the presently available anti-fungal azoles:
A. Fluconazole
B. * Itraconazole
C. Ketoconazole
D. Both Fluconazole & Ketoconazole
E. None of the above
276
Azole most likely to get into the cerebral spinal fluid:
A. Ketoconazole
B. Itraconazole
C. * Fluconazole
D. Both Ketoconazole & Itraconazole
E. None of the above
277
Given by i.v. administration, the drug of choice for nearly all life-threatening mycotic infections-usually used as the initial induction regimen
A. Ketoconazole
B. Itraconazole
C. Flucytosine
D. * amphotericin B
E. nystatin
278
Properties of tetracycline:
A. inhibitor of bacterial cell wall synthesis
B. * dug of choice in treating typhus
C. cleared primarily by the liver
D. All of the above
E. none of the above
279
Females often complaine: odorless, white or yellow cheesy discharge with itching
A. Chalymida
B. Syphilis
C. Chalmdyia
D. * Candidias
E. none of the above
280
A bacteriostatic antimicrobial that inhibits protein synthesis by blocking attachment of aminoacyi tRNA to the A site on the 30S bacterial ribosome
A. Aminoglycosides
B. Fluoroquinolones
C. Metronidazole
D. Sulfonamide
E. * Tetracycline
281
Bactericidal drugs that act by inhibiting DNA gyrase
A. Aminoglycoside
B. * Fluoroquinolones
C. Macrolides
D. Rifamycins
E. Sulfonamides
282
Binds to the 50 s bacterial ribosome and prevents the translocation step in protein synthesis
A. Aminoglycosides
B. Daptomycin
C. Linezolid
D. * Macrolides
E. Metronidazole
283
Binds to the 30s bacterial ribosome and blocks the initiation step in protein synthesis. May cause
miscoding
A. * Aminoglycosides
B. Daptomysin
C. Linezolid
D. Metronidazole
E. Rifampin
284
Inhibits the formation of folic acid by competing with PABA for dihydropteroate synthase
A. Metronidazole
B. Rifampin
C. * Sulfamethoxazole
D. Trimethoprin
E. Linezolid
285
Binds to specific receptors located in the bacterial cytoplasmic cell membrane and inhibits
transpeptidase enzymes, preventing cross linking of peptidoglycan chains in the cell wall
A. * Cephalosporins
B. Daptomycin
C. Linezolid
D. Metronidazole
E. Rifampin
286
Binds to the D-Ala-D-Ala terminal end of peptidoglycan side chains, resulting in inhibition of
transglycosylation and disruption of cross-linking in the cell wall
A. Chloramphenicol
B. Clavulanic
C. Imipenem
D. * Vancomycin
E. Metronidazole
287
Combination of these two drugs would produce an antagonistic (undesirable) antimicrobial effect
when treating an infection
A. Amoxicillin+ gentamicin
B. Cephalexin+ gentamicin
C. Sulfamethoxazole+ trimethoprim
D. * Tetracycline+ amoxicillin
E. Vancomycin+ gentamicin
288
Which form of resistance to the effect of antimicrobial drugs is specific to beta-lactam type
antibiotics
A. Decreased drug uptake
B. Methylation of the antibiotic’s receptor
C. Decreased metabolic activation
D. Altered amount of drug receptor
E. * Increased enzymatic destruction of the antibiotic
289
The combination of sulfamethoxazole and trimethoprim is a useful drug for treating urinary tract
infections. Bacterial resistance to trimethoprim occurs primarily by
A. Acetylation of trimethoprim
B. Acetylation of the drug receptor
C. Decreased bacterial uptake
D. * Upregulation of dihydrofolate reductase
E. Upregulation of beta-lactamase
290
Bacterial resistance to this drug results from a plasmid mediated mechanism that results in an altered
ribosomal binding site within the bacteria
A. Ampicillin
B. * Azithromycin
C. Penicillin
D. Piperacillin
E. Vancomycin
291
Which of the following antimicrobial drugs is bactericidal
A. Azithromycin
B. Doxycycline
C. Erythromycin
D. * Gentamicin
E. Tetracycyline
292
Which of the following antimicrobials is taken up into bacteria by an oxygen-dependent mechanim
and is therefore ineffective against anaerobic organisms
A. Cefoxitin
B. Clindamycin
C. * Gentamicin
D. Metronidazole
E. Ticarcillin
293
The drug of choice for methicillin-resistant Staphylococcus aureus is:
A. oxacillin
B. gentamicin
C. * vancomycin
D. piperacillin
E. None of the above
294
The drug of choice for Streptococcus pneumonia (pneumococcus) in a patient with no drug allergies
is
A. erythromycin
B. * penicillin
C. vancomycin
D. ceftriaxone
E. gentamicin
295
Which of the following is the general mechanism of action for erythromycin?
A. Inhibition of a metabolic enzyme
B. Inhibition of cell wall synthesis
C. * Disruption of protein synthesis
D. Inhibition of nucleic acid transcription and replication
E. None of the above
296
Which of the following is the general mechanism of action for fluoroquinolones?
A. Inhibition of a metabolic enzyme
B. Inhibition of cell wall synthesis
C. Disruption of protein synthesis
D. * Inhibition of nucleic acid transcription and replication
E. None of the above
297
Trimethoprim is often administered alongside a sulfonamide as a preparation known as cotrimoxazole. Which of the following sulfonamides is used?
A. * Sulfamethoxazole
B. Sulfathiazole
C. Sulfadoxine
D. Sulfadiazine
E. None of the above
298
Which of the following statements is true regarding the properties of benzylpenicillin?
A. It is a bacteriostatic agent.
B. It is active over a wide range of bacterial species.
C. It is resistant to ?-lactamases.
D. * Certain individuals may have an allergic response to it.
E. All of the above
299
What crucial feature of a penicillin is involved in its mechanism of action?
A.
B.
C.
D.
E.
Carboxylic acid
* beta-lactam ring
Acyl side chain
Thiazolidine ring
All of the above
300
The following structure is a first generation cephalosporin. What is the name of the structure?
A. Cefazolin
B. Cefoxitin
C. Cefuroxime
D. * Cefalexin
E. All of the above
301
What drugs are present in the preparation Augmentin?
A. Ticarcillin and clavulanic acid
B. Ampicillin and sulbactam
C. Ampicillin and clavulanic acid
D. * Amoxicillin and clavulanic acid
E. None of the above
302
Which of the following antibiotics is a macrolide?
A. Chloramphenicol
B. Doxycycline
C. * Erythromycin
D. Streptomycin
E. None of the above
303
Which of the following antibiotics is a tetracycline?
A. Chloramphenicol
B. * Doxycycline
C. Streptomycin
D. Erythromycin
E. None of the above
304
Which of the following drugs is associated with the reaction of hepatitis?
A. Valproic acid
B. Quinidine
C. * Isoniazid
D. Ethosuximide
E. All of the above
305
Drugs most often used in the treatment of petit mal epilepsy include:
A. ethosuximide
B. phenytoin
C. sodium valproate
D. All of the above
E. * ethosuximide & sodium valproate
306
The most specific agent for treatment of petit mal epilepsy is:
A. carbamazepine
B. phenytoin
C. gabapentin
D. * ethosuximide
E. primidone
307
Phenytoin may have adverse effects on the hematopoietic system. All of the following have been
reported EXCEPT:
A. macrocytic anemia
B. hepatosplenomegaly
C. folic acid deficiency
D. lymphadenopathy
E. * leukemia
308
The drug of choice in initiating chronic treatment of grand mal epilepsy is:
A. phenobarbital
B. ethosuximide
C. phenacemide
D. diazepam
E. * carbamazepine
309
Gingival hyperplasia is a common side effect of:
A. procainamide
B. * phenytoin
C. digitalis
D. indomethacin
E. guanethidine
310
Which of the following drugs (including metabolites) would achieve steady state blood level
concentrations earliest assuming a loading dose is not given?
A. phenytoin
B. primidone
C. * valproate
D. phenobarbital
E. ethosuximide
311
Which one of following is an appropriate therapeutic use for imipramine?
A. Insomnia.
B. Epilepsy
C. * Bed—wetting in children
D. Glaucoma.
E. Mania
312
MAO inhibitors are contraindicated with all of the following EXCEPT:
A. Indirect adrenergic agents, such as ephedrine.
B. Tricyclic antidepressants.
C. Beer and cheese.
D. * Aspirin.
E. Dopamine.
313
All of the following are observed in patients taking neuroleptic agents EXCEPT:
A. Sexual dysfunction.
B. * Increased blood pressure.
C. Altered endocrine function.
D. Constipation.
E. Orthostatic hypotension.
314
In the treatment of acute diarrhea
A. antibiotics are worst avoided in non-pathogenic diarrhea
B. antibiotics are worst avoided in viral gastroenteritis
C. oral rehydration should not be used for initial therapy
D. electrolytes and glucose should not be supplemented for initial therapy
E. * oral rehydration and electrolytes supplementation are required particularly in children and in the
elderly
315
Neostigmine will effectively antagonize skeletal muscle relaxation produced by:
A. * Tubocurarine
B. Succinylcholine
C. Diazepam
D. Baclofen
E. Nicotine
316
The reversible cholinesterase inhibitor indicated in the treatment of Alzheimer disease is:
A. * Rivastigmine
B.
C.
D.
E.
Levodopa
Neostigmine
Pyridostigmine
Acetylcholine
Direct-acting cholinomimetic drug that is lipid-soluble and often used in the treatment of glaucoma
317
is:
A. Acetylcholine
B. Timolol
C. Betaxolol
D. * Pilocarpine
E. Physostigmine
318
Atropine and scopolamine will block all the effects of acetylcholine listed below EXCEPT:
A. bradycardia
B. salivary secretion
C. bronchoconstriction
D. * skeletal muscle relaxation
E. miosis
319
Of the many types of adrenergic receptors found throughout the body, which is most likely
responsible for the cardiac stimulation observed following an intravenous injection (IV) of epinephrine?
A. beta1-adrenergic receptors
B. * alpha1-adrenergic receptors
C. alpha2-adrenergic receptors
D. alpha3-adrenergic receptors
E. None of the above
320
Which of the following drugs is a selective inhibitor of MAO type B?
A. Neostigmine
B. Selegiline
C. * Bromocriptine
D. Carbidopa
E. Entacapone
321
The drug of choice for the treatment of anaphylactic shock is:
A. * Epinephrine
B. Norepinephrine
C. Isoproterenol
D. Orciprenaline
E. Atropine
322
All the following statements are true concerning dobutamine EXCEPT that it:
A. is a selective agonist at alpha1-adrenergic receptors
B. * activates dopaminergic receptors in renal and mesenteric vascular beds
C. is used to increase cardiac output in patients with severe cardiac failure
D. must be given by IV administration
E. may cause tachycardia and anginal pain
323
Which one of the following drugs is a selective beta2-agonist?
A. Epinephrine
B. Norepinephrine
C. * Salbutamol
D. Isoproterenol
E. Dobutamine
324
The nonselective beta-adrenergic blocking agent that is also a competitive antagonist at alpha1adrenoreceptors is:
A. Timolol
B. Atenolol
C. Propranolol
D. Pindolol
E. * Carvedilol
325
Propranolol is indicated for use in patients with all the following conditions EXCEPT:
A. hypertension
B. angina pectoris
C. * bronchial asthma
D. migraine headache
E. supraventricular and ventricular arrhythmias
326
Which one of the following statements best describes the mechanism of action of benzodiazepines?
A. Benzodiazepines block GABA uptake
B. Their inhibition of GABA transaminase leads to increase of GABA
C. Benzodiazepines block glutamate receptors in the brain
D. * They increase frequency of opening of chloride ion channels that are coupled to GABAA receptors
E. They are direct-acting GABA receptor agonists in the CNS
327
The hypnotic drug facilitates the inhibitory action of GABA, but it lacks of anticonvulsant or muscle
relaxing properties, is:
A. Buspirone
B. Diazepam
C. Flurazepam
D. Phenobarbital
E. * Zolpidem
328
All the following benzodiazepines are biotransformed to active metabolites EXCEPT:
A. Alprazolam
B. Diazepam
C. * Oxazepam
D. Clorazepate
E. Chlordiazepoxide
329
Which one of the following is described as a competitive benzodiazepine receptor antagonist?
A. Pralidoxime
B. Bromazepam
C. Midazolam
D. Triazolam
E. * Flumazenil
330
All the following compounds are indicated for the treatment of psychoses EXCEPT:
A. Chlorpromazine
B. Risperidone
C. * Fluoxetine
D. Haloperidol
E. Clozapine
331
The preferred treatment of status epilepticus is IV administration of:
A. Chlorpromazine
B. * Diazepam
C. Succinylcholine
D. Ethosuximide
E. Selegiline
332
Carbidopa is useful in the treatment of Parkinson’s disease because it:
A. is a precursor of levodopa
B. is a dopaminergic receptor agonist
C. * prevents peripheral decarboxylation of levodopa
D. prevents breakdown of dopamine
E. promotes a decreased concentration of levodopa in the nigrostriatum
333
Which one of the following is an antidepressant agent that selectively inhibits serotonin (5HT)
uptake?
A.
B.
C.
D.
E.
Selegiline
* Fluoxetine
Maprotiline
Desipramine
Amitriptyline
334
Aspirin may be fatal in taken in sufficient quantity. The syndrome of fatal salicyate overdose in
children is characterized by
A. marked hypotermia secondary to an antipyretic effect
B. * fever, tinnitus
C. peripheral oedema
D. None of the above
E. All of the above
335
Common adverse reactions of corticosteroidal therapy are:
A. bradycardia, mental dullness
B. anorexia, polyuria
C. tachycardia, insomnia
D. * “moon face”, obese trunk
E. All of the above
336
The chemotherapeutic agent also used orally for severe forms of arthritis is
A. 5-fluorouracil (5-FU)
B. * methotrexate (Mexate)
C. cyclophosphamide (Cytoxan)
D. cisplatin (Platinol)
E. All of the above
337
What adverse effects are associated with chronic use of systemic corticosteroids?
A. Candidosis
B. * Gastrointestinal complications
C. Hypotension
D. Hypoglycemia
E. All of the above
338
For a 6-year-old child with fever, what NSAIDs would be preferred?
A. aspirin
B. * acetaminophen
C. indomethacin
D. ibuprofen
E. All of the above
339
Symptoms of salicylism:
A. Tinnitus
B. decreased hearing
C. vertigo
D. None of the above
E. * All of the above
340
Phases of inflammation:
A. chronic inflammation
B. immune response
C. acute inflammation
D. chronic inflammation & acute inflammation
E. * All of the above
341
Released at sites of tissue injury:
A. Kinins
B. complement components
C. cytokines
D. neuropeptides
E. * all of the above
342
Among these mediators: most significant increase in vascular permeability
A. Prostaglandins
B. Bradykinin
C. Serotonin
D. * leukotrienes
E. all of the above produce similar increases in vascular permeability
343
Mediator in acute inflammation, pain A. Histamine
B. Serotonin
C. Leukotrienes
D. * bradykinin
E. none of the above
344
Mediator of chronic inflammation (for example rheumatoid arthritis) A. Histamine
B. * interleukin-1
C. Bradykinin
D. Neuropeptides
E. Serotonin
345
Most nonsteroidal antiinflammatory drugs:
A. * inhibit prostaglandin biosynthesis
B. weak organic bases
C. probably increase production of free radicals
D. mainly associated with guanylyl cyclase interactions
E. all the above
346
Initial drug of choice for treating most articular and musculoskeletal disorders (because of low-cost,
safety, and efficacy)
A. * Aspirin
B. Diclofenac
C. Indomethacin
D. Phenylbutazone
E. Glucocorticoids
347
Concerning salicylates:
A. sodium salicylate an aspirin: equally effective as anti-inflammatory agents
B. aspirin: maybe more effective for analgesia
C. aspirin: hydrolyzed to acetic acid plus salicylate by tissue and blood esterases
D. urine alkalinization: increases free salicylate excretion
E. * all of the above
348
Acute attacks of gouty arthritis may occur early in treatment with allopurinol because:
A. allopurinol increases urate synthesis
B. * urate crystals move from tissue to plasma
C. allopurinol increases release of chemotactic factors
D. None of the above
E. All of the above
349
Rational indication(s) for allopurinol (Zyloprim, Purinol) administration:
A. chronic tophaceous gout; when tophi reabsorption more rapid with uricosuric agents
B. when probenecid or sulfinpyrazone cannot be used
C. presence of renal functional impairment
D. in a patient with recurrent renal stones
E. * all of the above
350
Effective in management of mild to moderate pain, when anti-inflammatory action is not necessary
A. penicillamine (Cuprimine)
B. sulfasalazine (Azulfidine)
C. * acetaminophen (Tylenol, Panadol)
D. etodolac (Lodine)
E. piroxicam (Feldene)
351
Probably the initial step in acute gouty arthritis attack:
A. polymorphonuclear leukocytes migration into the joint
B. increased numbers of mononuclear phagocytes (macrophages)
C. * urate crystals phagocytosis by synoviocytes
D. prostaglandin, lysosomal &, interleukin 1 release
E. None of the above
352
Probable direct effect of colchicine (mechanism of action):
A. direct membrane stabilization
B. * binds to intracellular tubulin -- preventing/reducing microtubule formation
C. decreases purine synthesis directly
D. prevents IL-1 release directly
E. none of the above
353
Reduces uric acid synthesis: for management of gout
A. colchicine
B. probenecid
C. sulfinpyrazone (Anturane)
D. * allopurinol
E. oxaprozin
354
Aspirin mechanism of action: anti-platelet effects:
A. promotes platelet aggregation
B. activates thromboxane synthesis
C. * snhibits thromboxane synthesis
D. all of the above
E. none of the above
355
Factors accounting for rheumatoid arthritis incidence:
A. Genetics
B. Climate
C. Urbanization
D. * All of the above
E. None of the above
356
At low doses required to inhibition of thymidylate synthase, an enhanced adenosine release
A. Chloroquine
B. Gold
C. * methotrexate
D. cyclophosphamide
E. ketorolac
357
Aspirin and antipyresis:
A. aspirin -- best available drug for reducing fever (in the absence of contraindications to its use
recurrent
B. aspirin -- more effective in lowering elevated temperature than normal body temperature
C. aspirin-induced temperature reduction is caused by vasodilation
D. * All of the above
E. None of the above
358
Phases of inflammation:
A. acute inflammation
B. the immune response
C. chronic inflammation
D. * All of the above
E. None of the above
359
Advantages of other NSAIDs compared with aspirin:
A.
B.
C.
D.
E.
generally less expensive
less gastric irritation
potentially better compliance (e.g. naproxen, sulindac)
generally less expensive & less gastric irritation
* less gastric irritation & potentially better compliance (e.g. naproxen, sulindac)
360
Aspirin:
A. inhibits prostaglandin synthase
B. inhibits cyclooxygenase
C. decreases prostaglandin formation
D. decreases thromboxane A2 formation
E. * All of the above
361
Mediators of chronic inflammation in rheumatoid arthritis:
A. IL-1
B. IL-2
C. IL-3
D. TNF alpha
E. * All of the above
362
Effective in managing acute gouty arthritis and ankylosing spondylitis; also accelerates closure of
patent ductus arteriosus in premature infants:
A. Gold
B. Ketorolac
C. Phenylbutazone
D. Methotrexate
E. * Indomethacin
363
Approximate prevalence of rheumatoid arthritis:
A. * 1 %
B. 5 %
C. 7 %
D. 10 %
E. 15 %
364
Analgesic effects of aspirin:
A. peripheral action (inflammation)
B. subcortical site of action
C. activates thromboxane synthesis
D. * peripheral action (inflammation) & subcortical site of action
E. subcortical site of action & activates thromboxane synthesis
365
NSAID primarily promoted as an analgesic, not as an anti-inflammatory agent:
A. Piroxicam
B. Ibuprofen
C. Naproxen
D. * Ketorolac
E. Sulindac
366
Drug associated with the hepatic/renal toxic metabolite: N-acetyl-p-benzoquinone
A. Diclofenac
B. Meclofenamate
C. Indomethacin
D. * Acetaminophen
E. Aspirin
367
Typically associated with B cell proliferation and differentiation into antibody-secreting plasma cells
A. tumor necrosis factor beta
B. interferon-gamma
C. * IL-4 (interleukin 4)
D. IL-2 (interleukin 2)
E. all of the above
368
Which of the following describes an agonist?
A. substance that brings about a change in biologic function through its chemical action
B. * A drug that binds to a receptor and stimulates cellular activity
C. A drug that binds to a receptor and inhibits or opposes cellular activity
D. A drug directed at parasites infecting the patient
E. None of the above
369
What determines the degree of movement of a drug between body compartments?
A. Partition constant
B. Degree of ionization
C. pH
D. Size
E. * All of the above
370
Which of the following is NOT a protein target for drug binding?
A. * Side of action (transport)
B. Enzymes
C. Carrier molecules
D. Receptors
E. Ion channels
371
Which of the following is an example of a drug acting directly through receptors?
A. Protamine binds stoichiometrically to heparin anticoagulants
B. Adrenergic beta blockers for thyroid hormone-induced tachycardia
C. * Epinephrine for increasing heart rate and blood pressure
D. Cancer chemotherapeutic agents
E. Mannitol for subarachnoid hemorrhage
372
What type of drug is propranolol (Inderal)?
A. Anticonvulsive
B. * Antihypertensive
C. Antinauseant
D. Antihistamine
E. Antipyretic
373
What type of drug is amlodipine?
A. * Ca++channel blocker
B. Adrenergic beta-blocker
C. ACE inhibitor
D. Blocks beta-receptors in heart myocardium
E. Diuretic
374
Which of the following is considered the class?
A. Propranolol
B. Inderal
C. * Adrenergic beta-blocker
D. “off label” use
E. Blocks beta-receptors in heart myocardium
375
Which of the following cases would be contraindicated for propranolol (Inderal)?
A. Hypertension
B. Essential tremor
C. Angina
D. Tachycardia
E. * Asthma
376
Which of the following adverse effects (side-effects) is NOT commonly seen with cholinergic
antagonists?
A. Blurred vision
B. Confusion
C. * Miosis
D. Constipation
E. Urinary retention
377
The drug chloramphenicol (Chloromycetin) is risky for which of the following?
A. * Neonates
B. Geriatric patients
C. Adult males
D. Obese patients
E. Congestive heart failure patients
378
Which of the following are the two modifying factors that contribute to why women have higher
blood peak concentrations of alcohol than men when consuming equivalent amounts?
A. Lower blood volume & increased hormones
B. Lower fat content & more gastric alcohol dehydrogenase (ADH)
C. Higher fat content & more gastric alcohol dehydrogenase (ADH)
D. Lower fat content & less gastric alcohol dehydrogenase (ADH)
E. * Higher fat content & less gastric alcohol dehydrogenase (ADH)
379
Pharmacokinetics is the effect of the ____ and pharmacodynamics is the effect of the ____.
A. Drug on a drug; Body on the drug
B. Body on the drug; Drug on a drug
C. Drug on the body; Body on the drug
D. * Body on the drug; Drug on the body
E. Drug on a drug; Drug on a drug
380
Which of the following is NOT an action of the body on a drug?
A. Absorption
B. Distribution
C. Metabolism
D. Excretion
E. * Side effects
381
If a drug is 80% bound to blood elements or plasma proteins, what part is considered the free form?
A. * 20%
B. 40%
C. 50%
D. 80%
E. 100%
382
If a patient misses three doses of their daily drug, which of the following (in general) is the best
solution?
A. Take a 4x dose at the next dose time
B. Wait 3 more days (week total) then return to normal regimen
C. * Do nothing and continue normal regimen
D. Setup an appointment to have the patient evaluated
E. Prescribe a higher dosage pill so missed doses will have less effect
383
Which of the following is NOT needed for drug bioequivalence?
A. Same active ingredients
B. Same strength or concentration
C. Same dosage form
D. Same route of administration
E. * Same side effects
384
For intravenous (IV) dosages, what is the bioavailability assumed to be?
A. 0%
B. 25%
C. 50%
D. 75%
E. * 100%
385
Although morphine is well-absorbed when administered orally, how much of the drug is metabolized
on its first pass through the liver?
A. * 90%
B. 70%
C. 50%
D. 30%
E. 10%
386
Which of the following is NOT a pharmacokinetic process?
A. Alteration of the drug by liver enzymes
B. Drug metabolites are removed in the urine
C. Movement of drug from the gut into general circulation
D. * The drug causes dilation of coronary vessels
E. The drug is readily deposited in fat tissue
387
Which of the following can produce a therapeutic response? A drug that is:
A. Bound to plasma albumin
B. Concentrated in the bile
C. Concentrated in the urine
D. Not absorbed from the GI tract
E. * Unbound to plasma proteins
388
A patient presents with an overdose of acidic Aspirin. The drug ____ can be given to ____ the pH of
the urine and trap the Aspirin, preventing further metabolism.
A. * NaHCO3; Increase
B. NaHCO3; Decrease
C. NH4Cl; Increase
D. NH4Cl; Decrease
E. None of the above
389
A patient presents with an overdose of alkaline Codeine. The drug ____ can be given to ____ the pH
of the urine and trap the Codeine, preventing further metabolism.
A. NaHCO3; Increase
B. NaHCO3; Decrease
C. NH4Cl; Increase
D. * NH4Cl; Decrease
E. None of the above
390
Bioavailability (F) is the fraction or percentage of administered drug that reaches the systemic
circulation via a given route as compared to what route?
A. Oral
B. * IV (intravenous)
C. IO (intraosseous)
D. CSF (cerebrospinal fluid)
E. Whatever route attains the target drug concentration in plasma (CT)
391
What organ is responsible for metabolism in the “first pass effect”?
A. Brain
B. Heart
C. Kidney
D. * Liver
E. Spleen
392
Which of the following routes of administration is the most convenient, although may have a
bioavailability anywhere from 5-100%?
A. * PO (oral)
B. IV (intravenous)
C. IM (intramuscular)
D. SQ (subcutaneous)
E. Transdermal
393
Which of the following enteral administration routes has the largest first-pass effect?
A. SL (sublingual)
B. Buccal
C. Rectal
D. * Oral
E. None of the above
394
Which of the following administration routes is not often used, is painful, and has a risk of infection
and adhesion?
A. EPI (epidural)
B. IA (intraarterial)
C. * IP (intraperitoneal)
D. IV (intravenous)
E. IO (intraosseous)
395
Which of the following is NOT an advantage of prolonged release medications?
A. Less frequent administration
B. Therapeutic effect overnight
C. Lower incidence of side effects
D. Patient compliance
E. * More fluctuation in plasma concentration
396
Which of the following would receive drug slowly?
A. Liver
B. Brain
C. * Fat
D. Muscle
E. Kidney
397
What type of drugs can cross the blood-brain barrier?
A. Large and lipid-soluble
B. Large and lipid-insoluble
C. * Small and lipid-soluble
D. Small and lipid-insoluble
E. None of the above
398
Basic drugs, such as lidocaine, bind primarily to which of the following plasma proteins?
A. alpha-1-fetoprotein (AFP)
B. Gc-Globulin (GcG)
C. Albumin
D. * alpha 1-acid glycoprotein (AAG)
E. Transcortin
399
A decrease in drug-protein binding will lead to which of the following?
A. Decrease in the unbound drug concentration
B. Increase in free drug
C. * Increase in rate of drug elimination
D. Decrease in volume of distribution
E. None of the above
400
Which of the following locations can accumulate lipid-soluble drugs, has little or no receptors, and
can hold distributed drugs like barbiturates?
A. Liver
B. Kidney
C. Brain
D. * Fat
E. Fetus
401
Which of the following locations has high blood flow and is a site of excretion?
A. Liver
B. * Kidney
C. Brain
D. Fat
E. Fetus
402
What is the approximate lag time for equilibration between maternal blood and fetal tissues?
A. 20 mins
B. * 40 mins
C. 1 hour
D. 2 hours
E. 6 hours
403
Elderly patients often have ____ muscle mass and thus a(n) ____ Vd (volume of distribution).
A. More; Increased
B. More; Decreased
C. Less; Increased
D. * Less; Decreased
E. None of the above
404
Most drugs are active in their ____ form and inactive in their ____ form.
A. * Non-polar; Polar
B. Polar; Non-polar
C. Water-soluble; Lipid-soluble
D. Lipid-insoluble; Water-insoluble
E. Neutral; Neutral
405
Which of the following metabolically active tissues is the principle organ for drug metabolism?
A. Skin
B. Kidneys
C. Lungs
D. * Liver
E. GI Tract
406
What is the goal of the P450 system (microsomes pinched off from endoplasmic reticulum)?
A. Metabolism of substances
B. Detoxification of substances
C. Increasing pH of compartments containing substances
D. Decreasing pH of compartments containing substances
E. * Metabolism of substances & Detoxification of substances
407
The direct-acting plasminogen activator is:
A. streptokinase
B. * urokinase
C. epsilon-aminocaproic acid
D. anistreplase
E. heparin
408
The antithrombotic drug which inhibits cyclooxygenase enzymes is:
A. prednisone
B. dipyridamole
C. tranexamic acid
D. * aspirin
E. ticlopidine
409
The mechanism of action of mini-dose heparin is correctly described by:
A. factor X is more sensitive to heparin than are other serine protease factors
B. mini-dose heparin is useful prophylactically rather than during active clot formation
C. by inhibiting factor X activation, a relative block of the clotting cascade develops, with less thrombin
formed
D. * All of the above
E. None of the above
410
The correct statement about fibrinolytic agents is:
A.
B.
C.
D.
E.
411
A.
B.
C.
D.
E.
412
A.
B.
C.
D.
E.
413
A.
B.
C.
D.
E.
414
A.
B.
C.
D.
E.
415
A.
B.
C.
D.
E.
416
A.
B.
C.
D.
E.
417
A.
B.
C.
D.
E.
418
A.
B.
C.
D.
E.
streptokinase acts directly to activate plasminogen
* urokinase acts directly to activate plasminogen
tissue-type specific plasminogen activator is more fibrin specific than streptokinase
side effects of t-PA include skin rash, allergic reactions, and development of antibodies
have been shown to increase mortality when given in the setting of an acute myocardial infarction
Following drugs stimulate erythrogenesis EXCEPT:
Iron dextran
Vitamine B12
* Methotrexate
Folic acid
Aspirin
Which drug does not influence leucopoiesis?
Filgrastim
* Erythropoetin
Doxorubicin
Methotrexate
All of the above
Iron deficiency anemia leads to:
pallor
fatigue
dizziness
exertional dyspnea
* All of the above
Tick the drug for parenteral iron therapy:
Ferrous sulfate
* Fercoven
Ferrous lactate
Ferrous fumarate
All of the above
Indicate the drug which increases absorption of iron from intestine:
Cyanocobalamin
Folic acid
* Ascorbic acid
Erythropoetin
All of the above
The drugs used for oral administration EXCLUDE:
Ferrous sulfate
* Fercoven
Ferrous lactate
Ferrous fumarate
None of the above
Pernicious anemia is developed due to deficiency of:
Erythropoetin
* Vitamin B12
Iron
Vitamin B6
Vitamin C
Select the drug used for pernicious anemia:
Ferrous lactate
* Cyanocobalamin
Iron dextran
Ferrous gluconate
Vitamin E
419
A.
B.
C.
D.
E.
420
A.
B.
C.
D.
E.
421
A.
B.
C.
D.
E.
422
A.
B.
C.
D.
E.
423
A.
B.
C.
D.
E.
424
A.
B.
C.
D.
E.
425
A.
B.
C.
D.
E.
426
A.
B.
C.
D.
E.
427
A.
B.
C.
D.
An adverse effect of oral iron therapy is:
Anemia
Thrombocytopenia
Headache
* Constipation
Hypertension
Choose the drug which contains cobalt atom:
Folic acid
Iron dextran
* Cyanocobalamine
Ferrous gluconate
Furosemid
Tick the drug used in aplastic anemia:
Fercoven
Cyanocobalamine
* Epoetin alpha
Folic acid
Methotrexate
Folic acid is recommended for treatment of:
* Megaloblastic anemia
Shock
Hypertension
Leukemia
None of the above
Select the drug of granulocyte colony-stimulating factor:
* Filgrastim
Methotrexate
Erythropoetin
Doxorubicin
Methotrexate
All of the following groups of drugs are for thrombosis treatment EXCEPT:
Anticoagulant drugs
* Antifibrinolitic drugs
Fibrinolitic drugs
Antiplatelet drugs
None of the above
Pick out the drug belonging to anticoagulants of direct action:
Aspirin
* Heparin
Dicumarol
Phenprocoumon
None of the above
Tick the drug used as an oral anticoagulant:
Heparin
Daltreparin
* Dicumarol
Enoxaparin
Fraxiparin
Which of the following drugs has low-molecular weight?
Dicumarol
* Enoxaparin
Phenprocoumon
Heparin
E. Aspirin
428
Indicate the drug belonging to antagonists of heparin:
A. Aspirin
B. Dicumarol
C. Dalteparin
D. * Protamine sulphate
E. Iron dextran
429
All of the following drugs are indirect acting anticoagulants EXCEPT:
A. Dicumarol
B. Warfarin
C. * Dalteparin
D. Phenindione
E. None of the above
430
Which of the following drugs belongs to coumarin derivatives?
A. Heparin
B. Enoxaparin
C. Dalteparin
D. * Warfarin
E. Aspirin
431
Heparin is effective when administred:
A. Orally
B. * Parenterally
C. Sublingually
D. All of the above
E. None of the above
432
All of these drugs are antiplatelet agents EXCEPT:
A. Aspirin
B. * Urokinase
C. Ticlopidine
D. Clopidogrel
E. Dipiridamol
433
Mechanism of aspirin action is:
A. Converts inactive plasminogen into active plasmin
B. * Inhibits COX and thus thromboxane synthesis
C. Enhances the interaction between antitrombin III and both thrombin and the factors involved in the
intrinsic clotting cascade
D. Inhibits the glycoprotein IIb/IIIa complex
E. All of the above
434
Which of the following drugs is an inhibitor of platelet glycoprotein IIb/IIIa receptors?
A. Aspirin
B. Clopidogrel
C. Ticlopidine
D. * Abciximab
E. Heparin
435
Which of the following drugs is fiibrinolytic?
A. Ticlopidine
B. * Streptokinase
C. Aspirin
D. Warfarin
E. Heparin
436
Fibrinolytic drugs are used for following EXCEPT:
A. Central deep venous thrombosis
B. Multiple pulmonary emboli
C. * Heart failure
D. Acute myocardial infarction
E. All of the above
437
Indicate the drug belonging to fibrinoliytic inhibitors:
A. * Aminocapronic acid
B. Ticlopidine
C. Streptokinase
D. Vitamin K
E. Vitamin C
438
Aminocapronic acid is a drug of choice for treatment of:
A. Acute myocardial infarction
B. * Bleeding from fibrinolytic therapy
C. Heart failure
D. Multiple pulmonary emboli
E. All of the above
439
Which of the following substances has its major activity as a saline cathartic?
A. sodium bicarbonate
B. methylcellulose
C. * sodium phosphate
D. castor oil
E. mineral oil
440
The correct statement regarding sucralfate:
A. pharmacologic action is to reduce gastric acid secretion by antagonizing gastrin
B. enhances N+-K+ ATPase
C. antagonizes acetylcholine
D. * most common side effects is constipation
E. increases gastric motility
441
Correct statement regarding metoclopramide:
A. central nervous system dopamine receptor agonist
B. peripheral blockage of acetylcholine at muscarinic synapse
C. decreases lower esophageal sphincter pressure
D. * adverse effects include dystonic or extrapyramidal effects
E. increases motility of colon
442
In general, mechanisms of laxation include:
A. adding bulk to the stool
B. increasing peristaltic activity
C. emulsifying aqueous and fatty substances with stool
D. lubricating the passage of stool
E. * All of the above
443
Appropriate indications for and/or uses of laxatives include:
A. prevent straining at the stool in patients with cardiovascular disease
B. bulk forming agents for diverticular disease
C. treatment of drug overdose
D. * All of the above
E. None of the above
444
Which of the following substances is most likely to cause systemic alkalosis?
A. * sodium bicarbonate
B. methylcellulose
C. sodium phosphate
D. castor oil
E. mineral oil
445
Saline cathartics, such as sodium sulfate or magnesium sulfate:
A. are safe in patients with renal failure
B.
C.
D.
E.
are readily absorbed from the gastrointestinal tract
are slower acting than bulk-forming laxatives
* act by increasing intestinal volume, hence stimulating peristaltic action
lower the surface tension of the feces to facilitate fecal hydration
446
Adverse effect(s) of laxatives:
A. electrolyte disturbances (hypernatremia, hypokalemia)
B. dehydration
C. spastic colitis with stimulant laxatives
D. gastrointestinal obstruction with bulk forming agents
E. * All of the above
447
In esophagitis, elevation of the head of the bed, abstinence from ethanol and tobacco, and small
frequent meals are all useful adjunctive therapeutic measures. Other useful therapy may include all of the
following EXCEPT:
A. omeprazole
B. metoclopramide
C. bethanechol
D. cimetidine
E. * amitriptyline
448
Drug which exerts anti-peptic ulcer effects through histamine-2 receptor antagonism:
A. sucralfate
B. ranitidine
C. metoclopramide
D. * omeprazole
E. misoprostol
449
The substance which is principally an emollient laxative is:
A. bran
B. methylcellulose
C. magnesium hydroxide
D. phenolphthalein
E. * mineral oil
450
Laxatives may aid in the treatment of portal systemic encephalopathy by:
A. increasing stool pH
B. increasing coliform bacteria
C. * decreasing protein contact time with GI mucosa
D. All of the above
E. None of the above
451
Agents of potential use in peptic ulcer disease include:
A. muscarinic antagonists
B. proton pump inhibitors
C. antacids
D. prostaglandins
E. * All of the above
452
Possible drug interactions: Aluminium hydroxide antacids tend to interfere with the gastrointestinal
absorption of:
A. cephalexin
B. penicillin G
C. erythromycin
D. chloramphenicol
E. * tetracycline
453
Prostaglandins E2 inhibit the secretion of gastric acid and stimulate the secretion of mucus. One
adverse effect limiting the wide use of prostaglandins for gastric disease is:
A. headache
B. thrombocytopenia
C. gynecomastia
D. * diarrhea
E. seizures
454
An agent which promotes defecation without increasing peristalsis is:
A. castor oil
B. phenolphthalein
C. * docusate sodium
D. cascara
E. milk of magnesia
455
Cimetidine reduces the clearance of all of the following EXCEPT:
A. theophylline
B. phenytoin
C. warfarin
D. * digoxin
456
Antacids having a relatively non-systemic effect include:
A. aluminum hydroxide
B. sodium bicarbonate
C. calcium carbonate
D. all of the above
E. * aluminum hydroxide and calcium carbonate
457
Which of the following substances has its major activity as an stimulant cathartic?
A. sodium bicarbonate
B. methylcellulose
C. sodium citrate
D. * castor oil
E. mineral oil
458
One mechanism to reduce gastric acid secretion is by blocking the K+-Na+ATPase pump in the
parietal cell. One drug that has this pharmacologic action is:
A. misoprostol
B. pirenzepine
C. * omeprazole
D. serotonin
E. isoniazid
459
The amount of sodium, phosphate or magnesium contained in an antacid should be assessed when
selecting an antacid for patients with:
A. renal insufficiency
B. congestive heart failure
C. ascites
D. * All of the above
E. None of the above
460
The concomitant administration of calcium and/or magnesium antacids to patients receiving one of
the tetracycline drugs may have which of the following effects upon the action of the tetracycline:
A. enhances the action
B. causes no significant change
C. * decreases the action
D. increases toxicity
E. suppresses hypersensitivity reactions
461
Which of the following substances has its major activity as a saline cathartic?
A. Sodium bicarbonate
B. Aluminium hydroxide
C. * Magnesium sulfate
D. Calcium carbonate
E. All of the above
462
In general, mechanisms of laxation include
A. adding bulk to the stool
B. increasing peristaltic activity
C. emulsifuing aqenons and fatty substances with stool
D. lubricating the passage of stool
E. * all of the above
463
Drug which exerts anti-peptic effects through histamine-2 receptor antagonism:
A. Denol
B. * Ranitidine
C. Omeprasole
D. Aluminium hydroxide
E. All of above
464
For treatment of heartburn patient regularly used some powder. After a week of drug using vomiting,
nausea, abdomen pain, fibrillation, shallow and slow breathing, alkalosis. What drug used patient?
A. * Natrii hydrocarbonas
B. Aluminium hydroxide
C. Magnesium sulfate
D. Omeprasole
E. Calcium carbonate
465
All these Drugs are cause obstipation Except:
A. Anticholinergic agents
B. Ca channel antagonists
C. Opioids
D. Tricyclic antidepressants
E. * Muscarinic agonists
466
Plant fiber (a laxative, purgative)
A. Decrease the bulk of the stools
B. increases the bowel transit time
C. * slowly distends the wall of the colon
D. increases the effective caloric content of the diet
E. takes down water and swells
467
Lactulose
A. is a monosaccharide
B. is broken down in the small intestine by bacteria
C. is build to unabsorbed organic anions which retain fluid
D. produces laxative effects after 2-3 hours
E. * is of particular value in the treatment of hepatic encephalopathy
468
Lactulose is of particular value in the treatment of encephalopathy:
A. * as it discourages the proliferation of ammonia producing organisms
B. as it increases the absorption of ammonia
C. as it decreases chronic portal hypertension
D. as it treatment fever
E. as it improves functions of CNS after absorption from the GIT
469
Magnesium sulfate
A. is a laxative which acts within 1-2 days
B. dilates the gallbladder and relaxes the sphincter of Oddi
C. decreases the secretion of cholecystokinin
D. decreases gastric, intestinal and pancreatic secretion
E. * should be given in dilute solution to a fasting individual
470
Senna alcaloids (anthraquinones, the sennosides A and B)
A. * act directly on the intramucosal plexus of the gut wall
B. take about 8 minutes to produce an effect
C. should be given to pregnant women
D. should be given to nursing mothers
E. can not induce diarrhea with excessive loss of water and electrolytes
471
Glycerol (in the form of rectal suppositories)
A. is useless if a rapid effect is required
B. * acts as a rectal stimulant due to local irritant action
C. cannot be used in children
D. rectal suppositories promote colonic evacuation in 30 hours
E. exerts severe diarrhea with loss of water and electrolytes
472
All these drugs are increasing intestinal transit time Except:
A. codeine (an opioid)
B. morphine (an opioid)
C. loperamide
D. diphenoxylate
E. * fysostigmin
473
Peptic ulcer disease
A. is an acute disorder
B. * characterized by frequent recurrences
C. comprises bones
D. the incidence of duodenal ulcers is four to five times lower than that of gastric ulcer
E. affects approximately 50% of the population
474
All these are major factors of known importance for the etiology of ulceration Except
A. acid-pepsin secretion
B. mucosal resistance to attack by acid and pepsin
C. * the age
D. effects of non-steroidal anti-inflammatory drugs
E. the presence of Helicobacter pylori
475
Acid secretion
A. is produced by endocrine cells in the gastric antrum cells
B. * is stimulated by gastrin
C. is inhibited by acetylcholine
D. is inhibited by histamine
E. is stimulated by prostaglandin E
476
Antacids:
A. * react with gastric acid to form a neutral salt
B. produce sweating
C. are Ineffective at healing duodenal ulcer
D. are very effective at healing gastric ulcers
E. its effect on acid secretion lasts for long (5 hours)
477
Antacids include:
A. balsalazide
B. orlistat
C. cimetidine
D. sucralfate
E. * aluminium salts (hydroxide, phosphate, glycinate)
478
Sodium bicarbonate (an antacid)
A. acts only locally
B. * excessive doses produce systemic alcalosis
C. produces carbon monoxide by reacting with hydrochloric acid
D. causes belching and distension of the large intestine
E. sodium intake need not to be considered in patients with hypertension or heart failure
479
Magnesium and aluminium salts
A. do undergo absorption so are effective locally
B. magnesium salts are constipating, aluminium salts may produce diarrhea
C. can not reduce the rate and extent of absorption of other drugs
D. aluminium salts should not be used with caution with any renal compromise
E. * magnesium and aluminium salts are taken 1-3 hours after meals and at bedtime
480
H2 lytics:
A. at least 4 days treatment is required to achieve healing
B. pain is relieved within 4 weeks treatment
C. include morphine , tramadol
D. * include nizatidine, ranitidine, famotidine
E. include pirenzepine
481
Ranitidine (ZANTAC)
A. has higher affinity for cytochrome P 450 than cimetidine
B. is less expensive than cimetidine
C. * is preferable to cimetidine in the elderly
D. has a similar profile of action to paracetamol
E. increases the plasma levels of theophylline
482
Pirenzepine
A. * is an M1 muscarinic receptor antagonist
B. can not cause mild difficulty with accomodation and dry mouth
C. can not alter the rate of absorption of other drugs if given concurrently
D. can be used in patients with concomitant glaucoma
E. can be used in patients with pyloric stenosis and prostatic enlargement
483
Omeprazol
A. is an irreversible stimulator of the proton pump
B. can be used only for healing gastric ulcer
C. Is for women only
D. is taken once weekly
E. * degrades in the presence of moisture. Capsules are supplied in special containers
484
Misoprostol
A. * is a synthetic analog of prostaglandin E1
B. produces gastric acid secretion
C. causes Stricture in the submucosa
D. decreases production of protective mucus
E. is indicated especially in pregnancy
485
Sucralfate
A. is less effective than cimetidine
B. is not so effective in symptom relief
C. antacids are contraindicated
D. contains aliminium, diarrhea can not be induced
E. * in severe renal failure accumulation is a potential hazard.
486
Cisapride
A. inhibits motility of the GIT
B. decreases rate of gastric emptying
C. is not used in gastroesophageal reflux
D. * is used in dyspepsia and delayed gastric emptying
E. is used in diarrhea
487
Metoclopramide is effective for:
A. preoperative vomiting
B. vestibular disturbances
C. motion sickness
D. headache
E. * facilitation of duodenal intubation and endoscopy
488
Drugs with a bacteriostatic effect in regular doses include:
A. tetracyclines
B.
C.
D.
E.
None of the above
sulfamethoxazole and trimethoprim (Sumetrolim)
erythromycin
* All of the above
489
The use of which of the following should be avoided in patients receiving monoamino-oxidase
inhibitor therapy:
A. cheese
B. imipramine (Melipramin)
C. None of the above
D. pethidine (Dolargan)
E. * All of the above
490
Penicillin administration is the appropriate therapy in which of the following complications of
syphilis?
A. * meningitis
B. aneurysm of the aorta
C. interstitial keratitisD. none of the above
E. the generalized paralysis of psychotic patients
491
Which of the following conditions is associated with true hematuria?
A. urinary tract tuberculosis
B. acute cystitis
C. renal infarction
D. none of the above
E. * all of the above
492
Which of the following signs can be attributed to an iron deficiency anemia?
A. a pale color of the sclera
B. hepatosplenomegaly
C. glossodynia (pain in the tongue)
D. * all of the above
E. none of the above
493
Haloperidol:
A. has marked extrapyramidal side effects
B. has a marked antiemetic effect
C. can be used to substitute for phenothiazine derivatives in patients who become jaundiced following
phenothiazine administration
D. * all of the above
E. none of the above
494
Indications of penicillamine therapy include:
A. systemic sclerosis
B. primary biliary cirrhosis
C. hemosiderosis
D. * all of the above
E. none of the above
495
The side effects of corticosteroids include:
A. a loss of collagen
B. decreased leukocyte migration
C. avascular bone necrosis
D. * all of the above
E. none of the above
496
Digitalis therapy:
A. is contraindicated in atrial tachycardia
B. * elongates the effective refractory period of the AV node
C. is likely to cause intoxication with a concomitant hyperkalemia
D. is contraindicated in cor pulmonale
E. is effective in hypertrophic obstructive cardiomyopathy
497
Possible therapeutic interventions in thyrotoxic crisis include:
A. the administration of beta-blockers
B. immediate propylthiouracil treatment
C. the administration of iodine
D. * all of the above
E. none of the above
498
Osteoporosis:
A. causes an elevation of the serum calcium concentration
B. typically causes elevation of the alkaline phosphatase activity
C. * causes pain in the bones
D. improves during bed rest
E. the response to calcium substitution therapy is usually positive
499
Barbiturates, if continuously administered can:
A. induce physical dependence
B. cause relaxation of skeletal muscles
C. cause ataxia
D. * all of the above
E. none of the above
500
Megaloblastic anemia is a possible side-effect of:
A. carbamazepine therapy
B. primidone (Sertan) therapy
C. methotrexate therapy
D. sulfamethoxazole and trimethoprim (Sumetrolim) therapy
E. * all of the above
501
Possible initial symptoms of diabetes mellitus include:
A. polydipsia
B. polyuria
C. severe pruritus
D. * all of the above
E. none of the above
502
In the nephrotic syndrome:
A. A generalized edema is present
B. the administration of steroids is always ineffective
C. favourable treatment includes the management of any underlying disease
D. * a generalized edema is present + favourable treatment includes the management of any underlying
disease
E. none of the above
503
In acute pyelonephritis:
A. * a common symptom is shaking chills
B. the absence of any lumbar pain excludes the diagnosis
C. an intravenous pyelogram is necessary for the diagnosis
D. to confirm the diagnosis, a hemoculture should routinely be made
E. all of the above
504
Immunosuppressive therapy is suitable in which of the following conditions?
A. psoriatic arthritis
B. * psoriatic arthritis + rheumatoid arthritis
C. persistent viremia
D. rheumatoid arthritis
E. all of the above
505
Which of the following statements about nitroglycerin are correct?
A. only topical application is effective in Raynaud's disease
B.
C.
D.
E.
it causes paroxysmal nocturnal dyspnea as a side-effect
it relieves the pain caused by diffuse esophageal spasms
* it might relieve pain in biliary colic
it alleviates the symptoms of bronchial asthma
506
Laboratory findings characteristic for early nephrotic syndrome include:
A. a serum albumin level which is lower than 25 g/1
B. a decreased fibrinogen level
C. an elevated serum cholesterol level
D. * a serum albumin level which is lower than 25 g/1 + an elevated serum cholesterol level
E. all of the above
507
The mechanism of anticancer action of fluorouracil is
A. Arrest in the G2phase of the cell cycle because of interference with the normal function of
topoisomerase enzymes
B. Cross-linking of the double stranded DNA and the resulting inhibition of DNA replication and
transcription
C. * Cytotoxicity resulting from metabolite that interferes with production of dTMP
D. Irreversible inhibition of dihydrofolic acid reductase
E. Selective inhibition of DNA polymerase
508
A drug that is used in the combination therapy of testicular cancer and is associated with
nephrotoxicity is
A. Beomycin
B. * Cisplatin
C. Etoposide
D. Leuprolide
E. Vinblastine
509
All of the following agents has been used in drug regime for the treatment of breast carcinoma.
Which one has specific activity in a subset of female breast cancer?
A. Anastrozole
B. Doxorubicin
C. Fluoxymesterone
D. Methotrexate
E. * Trastuzumab
510
This antimetabolite inhibits DNA polymerase and is one of the most active drugs in leukemias.
Although myelosuppression is dose limiting the drug may also cause cerebellar dysfunction, including
ataxia and dysartria.
A. Bleomycin
B. * Cytarabin
C. Dacarbazine
D. Doxorubicin
E. Etoposide
511
A widely used drug that supresses cellular immunity, inhibits prostaglandin and leukotriene
synthesis, and increases the catabolism of IgG antibodies is
A. Cyclophosphamide
B. Cyclosporine
C. Infliximab
D. Mercaptopurine
E. * Prednisone
512
Which of the anticancer drug causes hyperviscosity syndrome:
A. Vinblastine
B. * L-Asparaginase
C. 5-FU
D. Methotraxate
E. cyclophosphamide
513
A.
B.
C.
D.
E.
514
A.
B.
C.
D.
E.
515
A.
B.
C.
D.
E.
516
A.
B.
C.
D.
E.
517
A.
B.
C.
D.
E.
518
A.
B.
C.
D.
E.
519
A.
B.
C.
D.
E.
520
A.
B.
C.
D.
E.
521
A.
B.
C.
D.
What drug is mostly used for the treatment of chronic lymphatic leukemia?
* Chlorambucil
Prednisone
Cyclophosphamide
Myleran
Penicillin
Which of these groups is the most likely to experience anemia?
Men
* Women
Teenagers
Older adults
None of the above
How does anemia affect the body?
* The body is less able to absorb oxygen from the lungs
Blood becomes thin
Tissues retain fluids
None of the above
All of the above
The body needs an adequate supply of which of these in order for red blood cells to mature normally?
Oxygen
vitamin C
* vitamin B-12 and folic acid
Carbon dioxide
None of the above
All of the following may cause microcytic hypochromic anaemia, except:
* Vitamin B12 deficiency
Thalassemia
Iron deficiency
Gastrointestinal bleeding
Lactation
The sign of iron deficiency anaemia are all of the following except:
Increase in iron binding capacity
Decrease in serum ferritin level
Decrease in serum iron level
Decreased MCV
* Hyper-segmented neutrophils
The sign of megaloblastic anaemia is:
Increase in iron binding capacity
Decrease in serum ferritin level
Decrease in serum iron level
Decreased MCV
* Hyper-segmented neutrophils
The earliest sign of iron deficiency anaemia:
Increase in iron binding capacity
* Decrease in serum ferritin level
Decrease in serum iron level
Decreased MCV
Transferrin saturation
All of the following statements are true about sickle cell disease except
Vaso-occlusive crisis
Aplastic Crisis
* Hypertensive crisis
Sequestration Crisis
E. Hyperhemolytic crisis
522
Which is not seen in a chronic case of Sickle cell anemia:
A. Hepatomegaly
B. Pulmonaryhypertension
C. Cardiomegaly
D. Splenomegaly
E. * Peripheral neuropathy
523
Commonest acute presentation of sickle cell anaemia is:
A. Priapism
B. * Bone pain
C. Fever
D. Splenomegaly
E. Pulmonary hypertension
524
All of the following are the signs of anemia EXCEPT
A. Malaise
B. Dyspnea on exertion
C. Increasing cardiac output
D. Palpitations
E. * Dyspepsia
525
All of the following are the signs of anemia except
A. Malaise
B. Pallor
C. Increasing cardiac output
D. Palpitations
E. * Dysphagia
526
For diagnosis of anemia clinicians request all of the following except
A. Complete blood counts
B. Red blood cells
C. * Leukocyturea
D. Hemoglobin level
E. MCV
527
When the cause of anemia is not obvious, clinicians use all of the following tests except
A. Ferritin
B. Serum iron
C. * Fibrinogen
D. Serum vitamin B12
E. RBC folate level
528
Normocytic anemia are all of the following except
A. Acute blood loss
B. Anemia of chronic disease
C. Aplastic anemia (bone marrow failure)
D. Hemolytic anemia
E. * Vitamin B12 deficiency
529
The average blood volume for an adult is about
A. 45 liters
B. 450 ml
C. 5 gallons
D. * 5 liters
E. 3 liters
530
Each body function is correctly matched with the corresponding function of the blood on the right
EXCEPT
A. respiration - transports oxygen and carbon dioxide
B. * immune defense - platelet factors initiate clotting
C. acid-base balance - buffers acids and bases
D. thermoregulation - allows heat to escape from the body at the skin
E. Blood viscosity is due to the presence of the plasma proteins and erythrocytes.
531
Which statement concerning blood viscosity is CORRECT?
A. The viscosity of blood is 4.5 to 5.5 higher than the viscosity of water.
B. * Blood viscosity is due to the presence of the plasma proteins and erythrocytes.
C. The higher the blood viscosity the harder the heart has to work to move blood through the vessels
D. Anemia increases blood viscosity.
E. Hypocoaculation increases blood viscosity.
532
Other than water, the most common component of plasma is
A. Chloride
B. Urea
C. * Protein
D. Sodium
E. Calcium
533
Decrease in numbers of red blood cells or hemoglobin within red blood cells:
A. * Anemia
B. Erythrocytosis
C. Thrombocytosis
D. Leukemia
E. Leukocytosis
534
Which of the following statements concerning the diagnosis of pernicious anemia is true?
A. The presence of antiparietal-cell antibodies is diagnostic of pernicious anemia
B. Hematologic response to folate therapy alone rules out pernicious anemia as the cause of
megaloblasticanemia
C. Hyperkalemia may be a consequence of vitamin B12 therapy
D. Bone marrow examination would be expected to reveal marked depletion of erythrocyte precursors in
persons with untreated pernicious anemia
E. * Serum gastrin levels usually are elevated in persons with pernicious anemia
535
Which type of anemia has a 54-year-old man with artificial aortic value and fatigue, palpitations,
tachypnea on exertion, jaundice?
A. Iron deficiency anemia,
B. Megaloblastic anemia,
C. * Hemolytic anemia
D. Folic acid deficiency anemia,
E. B12 vitamin deficiency anemia
536
A 20 yrs adult presents with severe hypoplastic anemia. What is most effective treatment?
A. alpha-interferon
B. IL-2
C. ATG therapy
D. * Bone marrow transplantation
E. Prednisone
537
Which of signs is characteristic for hemolytic anemia?
A. * Jaundice
B. Pale lips
C. Sore mouth
D. Weakness
E. Fatigue
Situational tasks:
1. Patient entered the cardiologic department with the attack of angina pectoris, where a medical treatment
was appointed to him. After injection of medicines patient noted a strong headache and nausea. For what
2.
3.
4.
5.
6.
7.
8.
group of antianginal drugs is character such side effect?
A. Blockers of calcium channels
B. Beta – adrenergic blockers
C. Nitrates
D. * Vasodilators action on the coronal vessels
E. Beta – adrenergic stimulants
The patient suffers from hypertensive disease with bradiarrhythmia. What drug is necessary to prescribe?
A. * Platyphyllini hydrotartras
B. Ceftriaxone
C. Papaverinum
D. Paracetamol
E. Methyldopa
The 45 years old patient has diagnosis Angina pectoris. Cardiosclerosis. Arrhythmia. Hypertensive
disease. Choose the drug for treatment?
A. * Nebivolol
B. Suprastine
C. Potassium chloride
D. Strophantine
E. Lidocaine
At the patient after injecting drug for the medical treatment of hypertensive crisis tachycardia, and
orthosthatic hypotension in the vertical position has developed. What drug was injected?
A. Reserpine
B. * Clophelinum
C. Magnesium sulfate
D. Dibazolum
E. Verapamilum
To the patient with hypertensive disease was prescribed a drug from the adrenotropic group. After some
time the pressure normalized, but developed bradycardia (50 beats on minute) and the atrio-ventricular
block. What drug was prescribed?
A. * Propranolol
B. Papaverinum
C. Clophelinum
D. Mesatonum
E. Verapamilum
The 45 years old patient with the hypertensive disease has been taking antihypertensive drug during 4
days. The blood pressure normalized but somnolence and dry cough developed. What drug has been
taking the patient?
A. Clophelinum
B. Prazosinum
C. * Enalapril
D. Aspirine
E. Adrenaline
The 56 years old woman with hypertensive disease appealed to the doctor. Methyldopa was prescribed to
her. Indicate a mechanism of action of this drug.
A. * Violation of noradrenalin synthesis
B. Increasing of acetylcholine synthesis
C. Block of beta-adrenoreceptors
D. Block of alpha-adrenoreceptors
E. Depression of angiotensin converting enzyme activity
Patient with diagnosis of pheochromocytoma suffers from the high blood pressure which outgrows in
hypertensive crisis. What group of drugs will help in this situation?
A. Beta- adrenoblockers
B. Ganglionblockers
C. Sympatolitics
D. * Alpha-adrenoblockers
E. Calcium channels blockers
9. During assessment of a patient diagnosed with pheochromocytoma, the doctor reveales a blood pressure
of 210/110 mm Hg. The nurse would expect to administer which of the following medications?
A. nadolol
B. * phentolamine
C. dobutamine
D. verapamil
E. validolum
10. In the patient with the considerable peripheral edema the by turns using of dihlothiazid, ethacrynic acid
and Furosemide did not result in the considerable diuretic effect. The analysis of blood indicated the
considerable increasing of aldosteron level. Prescribe drug for treatment.
A. Mannitol
B. * Spironolactone
C. Clopamid
D. Triamterene
E. Amiloride
11. To the 55 years old man for the medical treatment of gout was prescribed etamid. What is the mechanism
of uric actions of the drug?
A. * Depression of reabsorbing of urinary acid
B. Increasing of secretion of urinary acid
C. Decreasing of production of urinary acid
D. Depression of activity of ksantinoxydase
E. Production of easy soluble salts
12. At the 46 years old patient with blink arrhythmia the edema of lungs began. What drug is necessary to
inject in the first turn?
A. * Furosemide
B. Triamterene
C. Verospirone
D. Amiloride
E. Euphylline
13. To the patient with edema of cardiac origin the doctor must prescribe diuretics. What drugs are
contraindicated in this situation?
A. Hypothiazid
B. Spironolactone
C. Furosemide
D. * Mannitol
E. Diacarb
14. In the patient with the considerable peripheral edema using of dihlothiazid did not result in the
considerable diuretic effect. The analysis of blood indicated the considerable increasing of aldosteron
level. Prescribe drug for treatment in this situation.
A. Mannitol
B. * Spironolactone
C. Ethacrynic acid
D. Furosemide
E. Diacarb
15. In the complex medical treatment of hypertensive disease a diuretic was prescribed to the patient. In a few
days the BP went down, but the signs of hypokaliaemia arose up. What drug could cause such
complication?
A. Triamterene
B. Clofeline
C. Enalapril
D. Spironolactone
E. * Furosemide
16. A patient with Type I diabetes presents with significant water retention and pulmonary congestion. Serum
creatinine and potassium were significantly elevated. The diagnosis was advanced renal failure. A diuretic
was given to treat water retention, but shortly thereafter the patient exhibited cardiac conduction changes
which progressed to heart block and cardiac arrest. The diuretic which was determined to cause this event
was probably:
A. furosemide (Lasix)
B. * triamterene (Dyrenium)
C. chlorothiazide (Diuril)
D. bumetanide (Bumex)
E. None of the above
17. Patient, in unconsciousness, entered the admission office. The skin was cold, the pupils was miotic, his
breathing was too hard and looked like Cheine-Stokes respiration; hypotension and overflowed urinary
bladder were marked. What was he poisoned with?
A. * Narcotic analgesics.
B. Tranquilizers.
C. Nonnarcotic analgesics.
D. M-cholinoblockers.
E. M-cholinomimetics.
18. The patient, who used prescribed preparation for a long time to treat acute respiratory diseases, suffered
from headache, vertigo, and noise in his ears, nausea, pain in epigastria region. What preparation can
cause such effects?
A. Midantan.
B. Vitamin C.
C. Naphthysini.
D. Bromhexini.
E. * Acetylsalicylic acid.
19. The patient, who suffered from irritability, sleeplessness, rapid fatigability and pain in the heart, consulted
a doctor. What preparation must be prescribed?
A. * Tincture Valerianae.
B. Sodium bromide.
C. Aminasin.
D. Droperidol.
E. Phenazepam.
20. A 5-year-old child had an attack of palpitation with nausea, dizziness, generalized fatigue. On ECG:
tachycardia with heartbeat rate of 220/min. Ventricle complexes are deformed and widened. P wave is
absent. What medication is to be prescribed to provide first aid?
A. * Lydocain
B. Seduxen
C. Novocainamides
D. Strophantin
E. Isoptin
21. A 46-year-old patient has ischemic heart disease, angina on exertion, II functional class. What is the drug
of choice in treatment of acute attack?
A. * Nitroglycerin sublingually
B. Platelet inhibiting agents (aspirin)
C. Spasmolitics (No-spa) IV
D. Digitalis IV
E. Sedative agents (Seduxenum) orally
22. A 52-year-old patient complains of intensive and prolonged retrosternal pains, decreased exercise
tolerance for 5 days. Which of the following groups is the most useful?
A. * nitrates
B. dyslipidemic drugs
C. diuretics
D. digitalis
E. ACE inhibitors
23. A 50-year-old patient was admitted to resusitation department with aqute myocardial infarction . Which
one of the following drugs is uneffective in this case?
A. Proranolol
B. Nitroglycerin
C. * Nifedipine
D. Phentanyl
E. Morfin
24. A 54-year-old patient, who suffers from ischemic heart disease, stable stenocardia II, heart insufficiency I,
has an attack of ventricular tachycardia, Ps 180/min, AP – 80/50 mm Hg. Choose a correct treatment:
A. * Lidocain, potassium chloride
B. Mezaton, strofantin
C. Verapamil, potassium chloride
D. Digoxin
E. ATP, cordaron
25. A 55-year-old men has stenocardia on exertion II. Taking of nitroglycerin potentiate a sever headache.
Which of the following drugs is the most useful in this case?
A. * Molsidomin
B. Amiodaron
C. Nifedipine
D. Propranolol
E. Verapamil
26. A 49-year-old patient has ischemic heart disease, angina on exertion, II functional class. What is the drug
of choice in treatment of acute attack?
A. No-spa IV
B. Aspirin
C. * Nitroglycerin sublingually
D. Digitalis IV
E. Seduxenum orally
27. A patient of 42 year has arterial hypertension with bradyarrhythmia. Which of the following drugs is
necessary to administer?
A. Klonidine
B. * Nifedipine
C. Diltiazem
D. Bisoprolol
E. Methyldopa
28. A 45-year-old women with diagnosis: Paroxysmal arrhythmia. Arterial hypertension II. Drug of choice
for stopping attack:
A. * Metoprolol
B. Nitroglycerin
C. Potassium chloride
D. Digoxin
E. Lidocain
29. A woman 56 years old with hypertension edema develops on lower extremities, moist wheezes in the
lower parts of lungs. What must be administered in the complex therapy of the patient?
A. Betaadrenomimetics,
B. * Diuretics,
C. Glucocorticoids,
D. Preparations of calcium,
E. M-cholinolitics.
30. A woman 51 years old has arterial hypertension. Which group of drugs may be used for her treatment?
A. Betaadrenomimetics,
B. * Angiotensin-converting enzyme inhibitors,
C. Glucocorticoids,
D. Preparations of calcium,
E. M-cholinolitics.
31. A man 36, years old, has arterial hypertension. Which group of drugs may be used for treatment?
A. Betaadrenomimetics
B. Preparations of calcium,
C. Glucocorticoids,
D. * Calcium_channel inhibitors,
E. M-cholinolitics.
32. Patients suffering from congestive heart failure will show signs and symptoms of peripheral
vasoconstriction, moist skin, pale complextion because of:
A. Na and water retention
B. decreased renin release
C. * increased sympathetic tone
D. decreased vagal tone
E. decreased aldosterone levels
33. A 59 year old patient with a history of systolic heart failure arrives in the ER with complaints of chest
palpitations, nausea and GI upset. His chart indicates that he is currently taking digoxin, captopril,
metoprolol and furosemide. A 12 lead ECG is recorded and reveals the presence of frequent runs of
ventricular bigeminy. His echocardiogram indicates an ejection fraction of 40%. His blood tests indicate a
serum potassium level of 4.0 mM, and an elevated digoxin level of 2.2 ng/ml. The most appropriate drug
for treating this patient's arrhythmia would be:
A. propranolol
B. quinidine
C. * lidocaine
D. verapamil
E. atropine
34. The 56 years old woman with hypertensive disease appealed to the doctor. Methyldopa was prescribed to
her. Indicate a mechanism of action of this drug.
A. Violation of noradrenalin synthesis
B. * alpha-adrenoreceptors blockers
C. Block of beta-adrenoreceptors
D. Depression of angiotensin converting enzyme activity
E. Increasing of acetylcholine synthesis
35. In the complex medical treatment of hypertensive disease a diuretic was prescribed to the patient. In a few
days the blood pressure went down, but the signs of hypokaliaemia arose up. What drug could cause such
complication?
A. Triamterene
B. Clofeline
C. Enalapril
D. Spironolactone
E. * Furosemide
36. The patient has cranial trauma. The brain edema threat developed in the postoperative period. To
prescribe drug for the removal of this complication.
A. Spironolactone
B. Papaverini hydrochloride
C. * Furosemide
D. Dihlothiazid
E. Diacarb
37. In the patient with the considerable peripheral edema the by turns using of dihlothiazid, ethacrynic acid
and furosemide did not result in the considerable diuretic effect. The analysis of blood indicated the
considerable increasing of aldosteron level. Prescribe drug for treatment.
A. Mannitol
B. * Spironolactone
C. Clopamid
D. Triamterene
E. Amiloride
38. At the 46 years old patient with blink arrhythmia the edema of lungs began. What drug is necessary to
inject in the first turn?
A. * Furosemide
B. Triamterene
C. Verospirone
D. Amiloride
E. Euphylline
39. An elderly male patient has essential hypertension, congestive heart failure, and type I insulin-dependent
diabetes. His congestive failure developed secondary to coronary vascular disease associated with
hyperlipidemia. What antihypertensive drug(s) may be most appropriate for this patient?
A. Chlorothiazide (Diuril)
B. * captopril (Capoten)
C. propranolol (Inderal)
D. metoprolol (Lopressor)
E. none of the above
40. An elderly female patient has essential hypertension, congestive heart failure, and type I insulindependent diabetes. His congestive failure developed secondary to coronary vascular disease associated
with hyperlipidemia. What antihypertensive drug(s) may be most appropriate for this patient?
A. chlorothiazide
B. * lisinopril
C. propranolol
D. metoprolol
E. all of the above
41. A patient presents with a blood pressure of 160/110 mm Hg. The patient has a history of coronary
vascular disease, resulting in angina, but has no evidence of congestive heart failure. The patient also has
asthma and has been treated mainly using terbutaline, by aerosol inhalation Propranolol (Inderal) was
prescribed to manage essential hypertension. Was this action appropriate?
A. Propranolol (Inderal) is appropriate because it will reduce heart rate and cardiac output. Negative
inotropism will help reduce the incidence of angina. It is an effective antihypertensive agent
B. Propranolol (Inderal) is inappropriate because it is only useful in mild hypertension; a better drug
would be minoxidil or hydralazine because they are more effective in lowering blood pressure
C. Propranolol (Inderal) is appropriate because it is an effective, low-cost antihypertensive. It will
augment the effects of terbutaline, an additional benefit
D. * Propranolol (Inderal) is inappropriate because its use is contraindicated in a patient with asthma.
E. All of the above
42. The patient with severe allergic bronchial asthma has been treated by oral drug during 7 months.
Hypertension, “moon face”, obese trunk, oedema, insomnia occur. What drugs does he used?
A. * Patient used one of orally used glucocorticoids, e.g. prednisolonum.
B. Patient used one of beta-agonists.
C. Patient used cromolynum.
D. Patient used euphyllinum
E. Patient used all above.
43. Patient 65 years old suffers from bronchial asthma. Adrenergic receptor activator is used for treatment,
After two weeks of management a pain near heart, palpitation. How can these side effects be prevent? It
is necessary to prescribe
A. * selective beta2-adrenergic receptor stimulator, for example salbutamolum.
B. glucocorticoids.
C. theophylline.
D. anticholinergics
E. cromolyn sodium.
44. An 81-year-old female with arteriosclerotic heart disease and pulmonary emphysema was found to have
significant bronchospasm. Her physician prescribed theophylline 400 mg every 12 hours. All of the
following toxicities may be observed EXCEPT:
A. cardiac arrhythmias
B. nausea and vomiting
C. agitation
D. * sodium and water retention
E. tremulousness
45. A patient complains of severe expiratory dyspnea, paroxysmal dry cough, general asthenia. This
condition lasts for 28 hours and is not benefite after usage of salbutamol. Data of examination: the patient
is staying and leaning against window-still, diffuse cyanosis add swelling of neck veins are observed,
weezes are heard on the distance, by percussion bundbox sound is heard, by auscultation – weakened
wesicular breathing, elongated expiration, difuse dry rales. Which medical preparation is the most
important for the patient?
A. beta-agonists,
B. beta-blockers;
C. Expectorants:
D. M-cholinolithics,
E. * Parenteral administrations of corticosteroids.
46. During auscultation of patient P., 60 years old, with chronic non-obstructive bronchitis, dry buzzling
rales above all parts of the lungs were heard as well as weakened vesicular breathing. What medical
preparation is it necessary to prescribe to the patient?
A. Diuretics;
B. Antitussive agents;
C. * Expectorants;
D. Broncholitics;
E. beta-blockers.
47. A 56-yr-old man wheezes and coughs. He has tried to give up smoking, but he finds it very difficult. He is
thin and healthy looking with a rounded chest. His breathing is noisy. His cough is unproductive. What
treatment has to be prescribed?
A. Amoxycillin
B. Prednisolone
C. * Ipratropium
D. Salbutamol
E. Bronchial lavage
48. A 20-yr-old woman presents with a week's history of fever, rigors arid productive rusty cough. CXR
shows right lower lobe consolidation. Prescribe treatment
A. Erythromycin
B. Co-trimoxazole
C. Prednisolone
D. * Amoxicillin
E. Salbutamol inhaler
49. Patients having a history of a severe, immediate reaction to penicillin:
A. may be given a cephalosporin without concern;
B. have a definite risk of reaction to any cephalosporin;
C. have a low risk of having a reaction to a broad spectrum antipseudomonal penicillin;
D. have a high risk of hypersensitivity to a broad spectrum anti-pseudomonal penicillin
E. * have a definite risk of reaction to any cephalosporin & have a high risk of hypersensitivity to a
broad spectrum anti-pseudomonal penicillin
50. A ninety two year-old female was brought to the emergency room following an episode of low blood
pressure and disorientation at the nursing home. She had been refusing food and drank little. She was
provided i.v glucose and was rehydrated. Over the next few days her condition worsened with increasing
periods of incoherence. She exhibited hyperventilation with respiratory alkalosis and pulmonary
congestion. Her body temperature was slightly subnormal and she was hypotensive (75/40 mm Hg) and
tachycardic. Her pulmonary congestion worsened as did her blood gases. Digoxin was administered.
Blood cultures indicated the present of E. coli, leading to a definitive diagnosis of E. coli septicemia.
Which drug(s) would be appropriate to treat the septicemia?
A. cefotaxime (Claforan)
B. ceftriaxone (Rocephin)
C. aztreonam (Azactan)
D. * all of the above
E. None of the above
51. A 23 years old patient is pregnant with a history of severe (anaphylactic) penicillin allergy. To prepare
her for an upcoming tooth extraction a doctor prescribes an antimicrobial medication that can be taken
prophylaxis 2 hrs prior to the procedure. This medication is
A. Cefaclor
B. Doxycycline
C. Erythromycin base
D. * Erythromycin stearate
E. Gentamycin
52. Circumstances associated with allergy to penicillins may include:
A. maculopapular or urticarial rash;
B. anaphylaxis;
C. anaphylactic reaction to penicillin skin-tests;
D. exposure to penicillins in food
E. * All of the above
53. A week after initiating clindamycin therapy a patient develops signs of a potentially fatal infection of the
colon. His tests reveal that his infection is due to clostridium difficile, to treat this new condition you
should initiate drug therapy with
A. a higher dose of clindamycin
B. gentamicin i.v
C. linezolid
D. * metronidazole
E. rifampin
54. Imipenem is a beta-lactam antibiotic which is neither a penicillin nor a cephalosporin. Correct statements
regarding imipenem include:
A. it covers an extremely broad spectrum of microorganisms;
B. it is very active against many gram negative rods;
C. resistant pseudomonas may emerge during treatment;
D. it should not be given to patients having a history of allergic reactions to penicillin
E. * All of the above
55. A patient with pulmonary disease is taking prednisone. A doctor begins her on phenytoin therapy for a
newly diagnosed seizure disorder. Two weeks later the patient returns to the clinic to see the doctor and
complains that her pulmonary disease has worsened since she began the phenytoin therapy. A likely
reason for this new change is that phenytoin:
A. interferes with absorption of the prednisone
B. stabilizes cell membranes, preventing the prednisone from diffusing to the site of action
C. * accelerates hepatic degradation of the prednisone
D. induce renal excretory pathways, accelerating urinary excretion of the prednisone
E. activates the asthma directly
56. A 21-year-old male has been brought to the emergency room unconscious three hours after ingestion of a
large dose of barbiturate. The immediate method of management would be:
A. administer concentrated dextrose intravenously
B. perform hemodialysis immediately with an artificial kidney
C. administer an analeptic drug such as pentylenetetrazol
D. * maintain a clear airway and artificially assist ventilation of air
E. exchange is unsatisfactory
57. Which of the following are true regarding cephalosporins?
A. "third generation" cephalosporins are generally more active against gram-negative organisms;
B. cephalosporins may depress beta-lactamase in certain organisms (Enterobacter, pseudomonas,
serratia) causing production of the enzyme to increase markedly;
C. the enzyme in part B above binds to the cephalosporin;
D. enterococcus is never reliably sensitive to any available cephalosporin
E. * All of the above
58. The administration of one drug may stimulate the metabolism of another drug (enzyme induction).
Phenobarbital is an example of such an enzyme inducer. The metabolism of which of the following drugs
may be affected by phenobarbital administration?
A. warfarin
B. phenytoin
C. digitoxin
D. * All of the above
E. None of the above
59. In the differential diagnosis of hirsutism, drug-induced hirsutism must be considered. A drug which is
known to include hirsutism as a side- effect is:
A. thallium
B. heparin
C. cephalosporins
D. * phenytoin
E. diazepam
60. A patient comes to the office complaining of sore, enlarged gums. She is currently taking medication for
hypertension and phenobarbital and phenytoin for seizures. The fibrous hyperplasia is most probably due
to:
A. * phenytoin
B. excessive brushing of teeth
C. propranolol
D. staphylococci infection of gums
E. phenobarbital
61. Appropriate indications for and/or uses of laxatives include:
A. * All are correct
B. prevent straining at the stool in patients with cardiovascular disease;
C. bulk forming agents for diverticular disease;
D. treatment of drug overdose;
E. None of the above
62. In esophagitis, elevation of the head of the bed, abstinence from ethanol and tobacco, and small frequent
meals are all useful adjunctive therapeutic measures. Other useful therapy may include all of the
following EXCEPT:
A. omeprazole
B. metoclopramide
C. bethanechol
D. cimetidine
E. * amitriptyline
63. What drug used patient? For treatment of heartburn patient regularly used some powder. After a week of
drug using vomiting, nausea, abdomen pain, fibrillation, shallow and slow breathing. Biochemical
examination show alkalosis.
A. * Natrii hydrocarbonas
B. Aluminium hydroxide
C. Magnesium sulfate
D. Omeprasole
E. Calcium carbonate
64. A doctor is caring for a client with a diagnosis of rheumatoid arthritis who is receiving sulindac (Clinoril)
150 mg po twice daily. Which finding would indicate to the doctor that the client is experiencing a side
effect related to the medication?
A. * nausea
B. photophobia
C. fever
D. tingling in the extremities
E. All of the above
65. To the patient with gout the doctor prescribed allopurinol. What pharmacological property of allopurinol
is important in this situation?
A. * Competitive inhibition of xantinoxydase.
B. Increasing of evacuation of nitrogen substances.
C. Acceleration of catabolism of pirimidinic nucleotides
D. Decreasing of reutilization of pirimidinic nucleotides
E. Increasing of synthesis of nucleic acids
66. At the 42 years old man with gout the concentration of urinary acid in the blood increased. For the decline
of level of urinary acid allopurinol was prescribed to him. What enzyme is allopurinol competitive
inhibitor of?
A. * Xantinoxydase
B. Adeninphosphoriboziltransferase
C. Hypoksantinphosphoriboziltransferase
D. Guanindesaminase
E. Adenosindesaminase
67. The patient, who suffered from irritability, sleeplessness, rapid fatigability and neurosis has been
diagnosed. What agents can be prescribed?
A. Anapriline.
B. Sodium chloride
C. Aminasine.
D. Atropine.
E. * Sibasone.
68. The patient, who suffered from anxiety, fear, hesitancy Sibasone has been prescribed. What is the
mechanism of its tranquilizing activity?
A. * Interaction with benzodiazepine receptors.
B. Interaction with adrenergic receptors.
C. Interaction with cholinergic receptors.
D. Interaction with serotonin receptors.
E. Interaction with dophaminergic.
69. The patient, who suffered from hyper excitability, irritability, sleeplessness was prescribed phenazepam.
What is the mechanism of its action?
A. * Stimulation of benzodiazepine receptors.
B. Stimulation of GABA (gamma amino butyric acid) receptors.
C. Stimulation of beta-adrenergic receptors.
D. Stimulation of M-cholinergic receptors.
E. Stimulation of H-cholinergic receptors.
70. The patient, suffered from schizophrenia, was prescribed aminazine for delusion and hallucinations
reducing. What is the mechanism of its antipsychotic action?
A. * Block adrenergic dophaminergic receptors in CNS.
B. Stimulate adrenergic dophaminergic receptors in CNS.
C. Stimulate cholinergic receptors in CNS.
D. Block cholinergic receptors in CNS.
E. Inhibition of re-uptaking of MAO.
71. The patient instead aminasine was prescribed another neuroleptic. What drug belongs to this group?
A. * Haloperidole.
B. Phentanile.
C. Paracetamol
D. Sibasone.
E. Morphine.
72. The physician monitors a patient taking an antipsychotic medication for side effects, including:
A. Hyper salivation
B. Hypetension
C. Diarrhea
D. * Extrapyramidal syndrome
E. Cramps
73. At the hands of nurse, working at the psychiatric unit during two years symptoms of neurodermatitis
appears. What drug can cause such adverse effect?
A. * Aminazine
B. Morphine.
C. Paracetamol.
D. Atropine.
E. Diasepam
74. For the patient with depression was prescribed Nialamide. The physician informed patient about
necessity to remove from the diet during of medical treatment:
A. Apples
B. * Cheese
C. Potatoes
D. Cabbages
E. Pears
75. To the patient for the removal of depressed syndrome was prescribed inhibitor of monooxygenase
(MAO). What food products are necessary to remove from the patient’s diet?
A. * Hard sorts of cheese, bananas, peanut,
B. Cabbages, cucumbers, tomatoes,
C. Millet, buckwheat,
D. milk porridges,
E. Honey, fruits
76. The old patient complains to headache, dizziness, rapid fatigue, worsening a memory. In anamnesis is
cranial-cerebral trauma. What pharmacological group is necessary to prescribe?
A. Hypnotics,
B. Neuroleptics,
C. Tranquilizers,
D. * Nootrops,
E. Analgetics
77. The physician notes lithium on a patient's drug history upon admission. The physician would suspect that
this patient suffers from:
A. Obesity
B. Abstinent syndrome
C. Renal disorder
D. * Manic episodes
E. All of the above
78. The problem of skeletal muscle contraction exist at child after poliomyelitis. What medicine is possible
to prescribe?
A. * Galantamine hydrobromide
B. Platyphylline
C. Methacine
D. Atropine sulfate
E. Tubacurarine
Tests questions to figures:
1.
A.
B.
C.
D.
E.
2.
A.
B.
C.
D.
E.
3.
A.
B.
C.
D.
E.
4.
A.
B.
C.
D.
E.
5.
A.
B.
C.
D.
E.
6.
A.
B.
C.
D.
E.
7.
A.
B.
C.
D.
E.
8.
A.
B.
C.
D.
Specify the group belonging of the drug anaprilin, shown in Figure 1:
*Beta-adrenoblocker
Alpha-adrenoblocker
Alpha, beta-adrenomymetic
Sympatholytic
None of the above
Specify the indications for use of the drug anaprilin indicated in Figure 1:
Coronary heart disease,
hypertension,
heart rhythm disturbances
*all of the above
none of the above
Specify the daily dose of the drug anaprilin indicated in Figure 1:
*60-80 mg / day
120-240 mg / day
2-4 mg / day
0,5-1,5 g / day
5-10 mg / day
Specify the way of using of the drug noradrenalin, shown in Figure 2:
*IV
IM
Oral
SQ
IQ
Noradrenaline (Figure 2) normally produces effects such as:
increased heart rate,
increased blood pressure,
dilation of pupils,
dilation of air passages in the lungs
*all of the above
What indications for using of the drug Noradrenaline (Figure 2)?
*Acute hypotension
Hypertension
Heart failure
Angina pectoris
All of the above
What indications for using of the drug adelphan, shown in Figure 3?
*Hypertensive diseases
Diabetes mellitus
Hypertensive emergency
Heart failure
Arrhythmia
The drug noradrenalin, shown in Figure 2, has _______________effect:
Vasodilating
*Vasoconstricting
Bronchodilating
Bronchospastic
E.
Analeptic
9.
Important contraindications to norepinephrine, shown in Figure 2, are____
A.
concomitant exposure to halothane anesthesia,
B.
hypersensitivity to the drug
C.
pregnancy
D.
lactation
E.
*all of the above
10.
Specify the group belonging of the drug doxazosin (Cardura), shown in Figure 4
A.
Beta-adrenomymetic
B.
*Alpha-adrenoblocker
C.
Beta-adrenoblocker
D.
Sympatholytic
E.
Alpha, beta-adrenomymetic
11.
Doxazosin, shown in Figure 4, is used
A.
to treat hypertension
B.
to improve urination in men with benign prostatic hyperplasia
C.
to treat hypotension
D.
*to treat hypertension & to improve urination in men with benign prostatic hyperplasia
E.
None of the above
12.
Specify common side-effects of Doxazosin, shown in Figure 4:
A.
Dizziness
B.
Headache
C.
Tiredness
D.
Swelling of the feet or lower legs
E.
*All of the above
13.
Specify the group belonging of the drug propranolol, shown in Figure 6:
A.
Beta-adrenomymetic
B.
Alpha-adrenoblocker
C.
*Beta-adrenoblocker
D.
Sympatholytic
E.
Alpha, beta-adrenomymetic
14.
Choose the correct statements about the drug arduan shown in Figure 9
A.
*long-acting non-depolarizing neuromuscular blocking agent
B.
For inhaled narcosis
C.
Non-inhaled narcotic agent
D.
Barbiturates derivatives
E.
Drug with pronounced stage of excitation
15.
Patient for induction of anesthesia intravenously entered drug thiopental, shown in Figure 10, then
came laryngospasm, reinforced by the salivation. The introduction of which the drug could be prevented
unwanted effects?
A.
Neostigmine
B.
Adrenaline HCl
C.
*Atropine sulfate
D.
Pirroxan
E.
Paracetamol
16.
Indicate which group the drug cyclodol, is shown in Figure 11 belongs to:
A.
Antiepileptic
B.
Antiparkinsonian
C.
Antipsychotics
D.
Tranquilizers
E.
Antidepressants
17.
The drug, represented on figure 13 (carbamazepine), belongs to:
A.
*Anti-epileptic
B.
C.
D.
E.
18.
A.
B.
C.
D.
E.
19.
A.
B.
C.
D.
E.
20.
A.
B.
C.
D.
E.
21.
A.
B.
C.
D.
E.
22.
A.
B.
C.
D.
E.
23.
A.
B.
C.
D.
E.
24.
A.
B.
C.
D.
E.
25.
A.
B.
C.
D.
E.
26.
Antiparkinsonian
Steroids
Antidepressants
Barbiturates
What is carbamazepine (figure 13) used for?
generalised tonic-clonic seizures and partial seizures
trigeminal neuralgia
bipolar affective disorder
none of the above
*all of the above
The drug, represented on figure 14- diphenin (phenytoin), belongs to:
Antiparkinsonian
Steroids
Antidepressants
Barbiturates
*Anti-epileptic
The drug, represented on figure 15 (trifluoperazine), belongs to:
Antiepileptic
Antiparkinsonian
*Neuroleptics
Tranquilizers
Antidepressants
The drug, represented on figure 16 (phenazepam), belongs to:
Antiepileptic
Antiparkinsonian
Neuroleptics
*Tranquilizers
Antidepressants
The drug, shown on figure 17 (gidazepam), belongs to:
Antiepileptic
Antiparkinsonian
Neuroleptics
*Tranquilizers
Antidepressants
The drug, shown on figure 18 (amitriptyline), belongs to:
Antiepileptic
Antiparkinsonian
Neuroleptics
Tranquilizers
*Antidepressants
The drug, shown on figure 19 (Azaphen), belongs to:
Antiepileptic
Antiparkinsonian
Neuroleptics
*Antidepressants
Tranquilizers
Specify indications for the drug (сyclodol), shown in Figure 11
*Parkinson's Disease
To prevent seizures
insomnia
alcoholism
Myasthenia gravis
Specify indications for the drug (teturam), shown in Figure 20
A.
B.
C.
D.
E.
27.
A.
B.
C.
D.
E.
28.
A.
B.
C.
D.
E.
29.
A.
B.
C.
D.
E.
30.
A.
B.
C.
D.
E.
31.
A.
B.
C.
D.
E.
32.
A.
B.
C.
D.
E.
33.
A.
B.
C.
D.
E.
34.
A.
B.
C.
D.
E.
Drug abuse
*Alcoholism
Acute alcohol poisoning
Acute poisoning with morphine
Barbituromaniya
With what are combined drug (teturam) indicated in Figure 20, in the treatment of alcoholism?
apomorphine
valerian
phenobarbital
morphine
*ethyl alcohol
Choose the correct statement for the drug nitrazepam, shown in Figure 21:
*Sleeping pills
Has hypotensive effect
To remove an attack of epilepsy
For the treatment of alcoholism
Anticonvulsive effect
Choose the correct statement for the drug diazepam, shown in Figure 22:
Hypotensive agent
Hypertensive agent
*Antiepileptic agent
Anticonvulsive agent
For the treatment of alcoholism
Choose the most typical side effect for the drug aspirin, shown in Figure 23
*Gastrointestinal ulcers
Ototoxicity
Nephrotoxicity
Hepatotoxicity
Edema
Choose the indications for the drug naloxon, shown in Figure 24
Chronic morphine poisoning
*Acute morphine poisoning
Poisoning with atropine
Poisoning with barbiturates
None of the above
Indicate the possible mechanism of analgesic effect of morphine (Figure 28):
*Violation of synaptic transmission in the ways of pain in the central nervous system
Antipyretic action
Inhibition of formatting of pain mediators
Decreasing of inflammations in peripheral tissues
Disturbance of prostaglandin synthesis
What agent administer for gastric lavage in case of poisoning with morphine (Figure 28)?
Sodium permanganate
Sodium hydrocarbonas
Potassium chloride
*Potassium permanganate
Sodium carbonas
Choose the correct statement for morphine, shown in Figure 28:
Indicated in headache
*Narcotic analgesic
Hypotensive drug
Indicated in toothache
Tranquilizers
35.
A.
B.
C.
D.
E.
36.
A.
B.
C.
D.
E.
37.
A.
B.
C.
D.
E.
38.
A.
B.
C.
D.
E.
39.
A.
B.
C.
D.
E.
40.
A.
B.
C.
D.
E.
41.
A.
B.
C.
D.
E.
42.
A.
B.
C.
D.
E.
43.
A.
B.
C.
D.
Show adverse side effect of anti-inflammatory agent butadion (Fig.25):
*Dyspeptic pains
Depressing of CNS
Hypothermia
Hypotension
Allergic reactions
The drug, shown on figure 26 (paracetamol), belongs to:
*Analgesic-antipyretic
Narcotic analgesic
Tranquilizers
Sedatives
Neuroleptics
Name indications for using of the agent (paracetamol), shown on figure 26:
Stimulation and analgesia during labor
*Headache
Angina pectoris
Acute poisoning with morphine
Couph
The drug, shown on figure 28 (morphine), belongs to:
Analgesic-antipyretic
*Narcotic analgesic
Tranquilizers
Sedatives
Neuroleptics
Choose the correct statement for morphine, shown in Figure 28:
*Depresses the breathing center
Antipyretic action
Non-narcotic analgesic
Sleeping pills
Indicated for treatment of poisoning with alcohol
Name indications for using of the agent (desmopressin), shown on figure 29:
*Diabetes insipidus
Diabetes mellitus
Heart failure
Acromegaly
None of the above
What disease is characterized by ,,moon face" (figure 32)?
*Cushing's disease
Obesity
Feochromocytoma
Glomerulonephritis
Hyperparathyroidism
In which of the listed diseases Xenical is used (Fig. 34)?
Thyrotoxicosis
*Obesity
Acromegaly
Hyperparathyroidism
Hypoparathyroidism
For the treatment of which disease parlodel is used (Fig.35)?
*Hypophysis adenoma
Feochromocytoma
Obesity
Hypoparathyroidism
E.
Hypothyroidism
44.
For the prevention of which complication of diabetes mellitus is used berlipril, shown in Fig.41?
A.
*Diabetic nephropathy
B.
Diabetic angiopathy
C.
Diabetic cardiomyopathy
D.
All of the above
E.
None of the above
45.
In Fig.42 shows berlithion which belongs to alpha-lipoic acid. Which of the following drugs does not
belong to this group?
A.
*Cavinton
B.
dialipon
C.
Espa-lipon
D.
Tiohamma-turbo
E.
None of the above
46.
What group of drugs belongs diroton (lisinopril) (Fig.44)?
A.
*ACE inhibitor
B.
Betablocker
C.
Ca++channel blocker
D.
Diuretic
E.
Angiotensin II blocker
47.
For what purpose enap (enalapril) (Fig.45) is used in diabetes mellitus?
A.
*Prevention of nephropathy
B.
Treatment of hypertension
C.
Prevention of polyneuropathy
D.
Prevention of dismetabolic cardiomyopathy
E.
All of the above
48.
For the treatment of which complication of diabetes mellitus gabagamma is used (Fig.46)?
A.
*Polyneuropathy
B.
Nephropathy
C.
Angiopathy
D.
Encephalopathy
E.
Retinopathy
49.
What group of drugs belongs enalapril (Fig.45)?
A.
Betablocker
B.
Ca++channel blocker
C.
Diuretic
D.
*ACE inhibitor
E.
Angiotensin II blocker
50.
Insulin Actrapyd in the figure 48 refers to ________________ insulin:
A.
*The short-acting
B.
The long-acting
C.
The ultrashort-acting
D.
Combined action
E.
None of the above
51.
Insulin Humodarum R in the figure 49 refers to ________________ insulin:
A.
The ultrashort-acting
B.
*The short-acting
C.
The long-acting
D.
Combined action
E.
None of the above
52.
Insulin Mixtard in the figure 53 refers to ________________ insulin:
A.
The ultrashort-acting
B.
The short-acting
C.
The long-acting
D.
*Combined action
E.
None of the above
53.
To which group hypoglycemic drugs belongs drug glibenclamide (Fig.55)?
A.
*sulfonylurea derivatives
B.
biguanide derivatives
C.
thiazolidinedione derivatives
D.
Postprandial regulators of insulin secretion
E.
Drugs that reduce the absorption of glucose
54.
To which group hypoglycemic drugs belongs Gliquidone (Fig.56)?
A.
Drugs that reduce the absorption of glucose
B.
*sulfonylurea derivatives
C.
biguanide derivatives
D.
thiazolidinedione derivatives
E.
Postprandial regulators of insulin secretion
55.
To which group hypoglycemic drugs belongs metformine (Fig.57)?
A.
sulfonylurea derivatives
B.
*biguanide derivatives
C.
thiazolidinedione derivatives
D.
Postprandial regulators of insulin secretion
E.
Drugs that reduce the absorption of glucose
56.
To which group hypoglycemic drugs belongs repaglinide (Fig.58)?
A.
sulfonylurea derivatives
B.
biguanide derivatives
C.
thiazolidinedione derivatives
D.
*Postprandial regulators of insulin secretion
E.
Drugs that reduce the absorption of glucose
57.
To which group hypoglycemic drugs belongs Rosiglitazone (Fig.60)?
A.
sulfonylurea derivatives
B.
biguanide derivatives
C.
*thiazolidinedione derivatives
D.
Postprandial regulators of insulin secretion
E.
Drugs that reduce the absorption of glucose
58.
To which group hypoglycemic drugs belongs pioglitazone (Fig.63)?
A.
sulfonylurea derivatives
B.
biguanide derivatives
C.
*thiazolidinedione derivatives
D.
Postprandial regulators of insulin secretion
E.
Drugs that reduce the absorption of glucose
59.
To which group hypoglycemic drugs belongs acarbose (Fig.64)?
A.
sulfonylurea derivatives
B.
biguanide derivatives
C.
thiazolidinedione derivatives
D.
Postprandial regulators of insulin secretion
E.
*Drugs that reduce the absorption of glucose
60.
Female patient 27 years old (Fig.65) complains on weight loss (7 kg during last month), increased
irritableness, insomnia. On examination: reduced degree of nouruishment, pulse 92, rrhythmic character. AP
110/80 mm Hg. Observed ophthalmopathy. What disease is characterized by these symptoms
A.
Hypothyroidism
B.
*Diffuse toxic goiter
C.
Nodular goiter, euthyroidism
D.
Vascular dystonia
E.
Systemic lupus erhythematosus
61.
Which group of drugs dexamethasone belongs (Fig.69)?
A.
Mineralocorticoids
B.
*Glucocorticoids
C.
Analgesics
D.
Nonsteoidal anti-inflammatory drugs
E.
Adrenomymetics
62.
Describe the effect of the drug actovegin (Fig. 74)
A.
*Improves blood rheology
B.
Analgesic action
C.
Antiinflammatory action
D.
Cardioprotective effect
E.
Hypotensive action
63.
Specify the group belonging of the drug amoxicillin (Fig. 76)
A.
Antiviral
B.
*Antibiotic penicillin group
C.
Fluoruquinalone
D.
Cephalosporine
E.
None of the above
64.
The drug, represented on figure 5, belongs to:
A.
Beta-adrenomymetic
B.
*Opioid analgesic
C.
Beta-adrenoblocker
D.
Sympatholytic
E.
Alpha-, beta-adrenomymetic
65.
Patient with chronic bronchitis uses represented on figure 7 agent for a long period (Fig.7). What
adverse effects revealed in patient?
A.
Bradycardia
B.
Hypotension
C.
Independence
D.
Depressing CNS
E.
*Stimulating CNS
66.
Make the previous diagnosis in patient (Fig. 32).
A.
*Cushing's disease
B.
Obesity
C.
Feochromocytoma
D.
Glomerulonephritis
E.
Hyperparathyroidism
67.
Bromokriptin is used for what disease treatment? (Fig. 33)
A.
*Hypophysis adenoma
B.
Feochromocytoma
C.
Adrenal cortex adenoma
D.
Thyroid adenoma
E.
Hypophysis neuroblastoma
68.
Indicate the diagnosis in patient 48 years old (Fig.36)
A.
*Acromegaly
B.
Gout
C.
Osteoarthrosis
D.
Rheumatoid arthritis
E.
Endocrine arthropathy
69.
Indicate the diagnosis in patient 42 years old? (Fig.37)
A.
*Acromegaly
B.
Arthritis
C.
Osteoarthrosis
D.
Rheumatoid arthritis
E.
Endocrine arthropathy
70.
Female patient 27 years old (Fig.65) complains on weight loss (7 kg during last month), increased
irritableness, insomnia. On examination: reduced degree of nouruishment, pulse 92, rrhythmic character. AP
110/80 mm Hg. Cardiovasculary and pulmonary systems without changes. ÒÒH 0,1 mIU/l, Ò4 free 75
mmol/l. Peripheral edema not revealed. Make a diagnosis?
A.
*Diffuse toxic goiter
B.
Hypothyroidism
C.
Nodular goiter, euthyroidism
D.
Vascular dystonia
E.
Rheumatoid arthritis
71.
Female patient 38 years old (Fig.66) complains on emotional lability, nervousness, insomnia,
eyeglobes’ increasing, feeling of sand in eyes during last 2 years. What possible diagnosis?
A.
*Diffuse toxic goiter, endocrine ophthalmopathy
B.
Nodular goiter, endocrine ophthalmopathy
C.
Hypothyroidism, allergic conjunctivitis
D.
Diffuse goiter, euthyroidism
E.
Autoimmune thyroiditis, thyroid gland hyperfunction, endocrine ophthalmopathy.
72.
What additional investigations should be performed in patient (Fig.67) to confirm a diagnosis?
A.
C-reactive peptide
B.
Thyroid-stimulating hormone, glycolised hemoglobin
C.
*Thyroid-stimulating hormone, thyroid hormone T4
D.
X-ray of the scull
E.
Steroids
73.
Make the previous diagnosis (Fig.68)
A.
Thyroid gland is not enlarged
B.
Diffuse goiter 1 degree
C.
Hipothyroidism
D.
Diffuse goiter 1 degree
E.
*Nodular goiter
74.
Apparatus represented on Fig.70 is:
A.
*Glucometer
B.
Manometer
C.
Insulin pump
D.
Monitoring of insulin
E.
Tonometer
75.
Call way of using for represented agent (Fig.72)?
A.
*Oral
B.
Intravenous
C.
Intramuscular
D.
Subcutaneous
E.
External
76.
Represented on Fig.75 medicine is used for:
A.
*Cytoprotective effect on gastric mucosa
B.
Endobronchial protection
C.
Arterial pressure decreasing
D.
Reducing of edema
E.
Decreasing the sensitivity of CNS cough centers